You are on page 1of 104

Republic of the Philippines

POLYTECHNIC UNIVERSITY OF THE PHILIPPINES


OFFICE OF THE VICE PRESIDENT FOR BRANCHES AND CAMPUSES
MARAGONDON BRANCH

INSTRUCTIONAL MATERIALS

IN

GEED 10053
MATHEMATICS IN THE MODERN WORLD

Compiled by: Checked by:

Assoc. Prof. Cherry E. Angeles Assoc. Prof. Ayreenlee E. Resus


Mr. Welfredo B. Queremit Chairman
Faculty Committee on Instructional Materials

Date: _________________ Date: ___________________

Approved by:

Dr. Agnes Y. Gonzaga Assoc. Prof. Denise A. Abril


Head, Academic Programs Director

Date: _________________ Date: __________________


INTRODUCTION

Welcome to the Polytechnic University of the Philippines. This module will help you become an
effective learner and successfully meet the requirements of the course. You will discover that
you can learn in a very challenging way at your own pace.

THE POLYTECHNIC UNIVERSITY OF THE PHILIPPINES

VISION

PUP: The National Polytechnic University

MISSION

Ensuring inclusive and equitable quality education and promoting lifelong learning opportunities
through a re-engineered polytechnic university by committing to:

 provide democratized access to educational opportunities for the holistic development of


individuals with global perspective
 offer industry-oriented curricula that produce highly-skilled professionals with managerial
and technical capabilities and a strong sense of public service for nation building
 embed a culture of research and innovation
 continuously develop faculty and employees with the highest level of professionalism
 engage public and private institutions and other stakeholders for the attainment of social
development goal
 establish a strong presence and impact in the international academic community

PHILOSOPHY

As a state university, the Polytechnic University of the Philippines believes that:

 Education is an instrument for the development of the citizenry and for the enhancement
of nation building; and
 That meaningful growth and transmission of the country are best achieved in an
atmosphere of brotherhood, peace, freedom, justice and nationalist-oriented education
imbued with the spirit of humanist internationalism.

TEN PILLARS

Pillar 1: Dynamic, Transformational, and Responsible Leadership


Pillar 2: Responsive and Innovative Curricula and Instruction
Pillar 3: Enabling and Productive Learning Environment
Pillar 4: Holistic Student Development and Engagement
Pillar 5: Empowered Faculty Members and Employees
Pillar 6: Vigorous Research Production and Utilization
Pillar 7: Global Academic Standards and Excellence
Pillar 8: Synergistic, Productive, Strategic Networks and Partnerships
SUBJECT: GEED 10053: MATHEMATICS IN THE MODERN WORLD
PREPARED BY: Assoc. Prof. CHERRY E. ANGELES & WELFREDO B. QUEREMIT, LPT
ii
Pillar 9: Active and Sustained Stakeholders’ Engagement
Pillar 10: Sustainable Social Development Programs and Projects

SHARED VALUES AND PRINCIPLES

 Integrity and Accountability


 Nationalism
 Spirituality
 Passion for Learning and Innovation
 Inclusivity
 Respect for Human Rights and The Environment
 Excellence
 Democracy

POLYTECHNIC UNIVERSITY OF THE PHILIPPINES


MARAGONDON BRANCH

GOALS

 Quality and excellent graduates


 Empowered faculty members
 Relevant curricula
 Efficient administration
 Development – oriented researches
 State-of-the-art physical facilities and laboratories
 Profitable income – generating programs
 Innovative instruction
 ICT – driven library
 Strong local and international linkage

PROGRAM OBJECTIVES

The Bachelor in Secondary Education major in English aims to provide training and practice
that will enable students to:

1. Acquire basic and advance level of literacy, communication, critical thinking and other
skills relevant to higher learning.
2. Develop a comprehensive and meaningful knowledge and understanding of the
different subject disciplines.
3. Apply a wide range of teaching process skills that include curriculum development,
lesson planning, materials development, educational assessment and teaching
approaches.
4. Gain direct experience from the field through classroom observations, teaching
assistance and practice teaching.
5. Develop researchers with quality outputs.
6. Strengthen Community Engagement.

iii
SUBJECT: GEED 10053: MATHEMATICS IN THE MODERN WORLD
PREPARED BY: Assoc. Prof. CHERRY E. ANGELES & WELFREDO B. QUEREMIT, LPT
GEED 10053
MATHEMATICS IN THE MODERN WORLD
COURSE DESCRIPTION

COURSE TITLE : MATHEMATICS IN THE MODERN WORLD


COURSE CODE : GEED 10053
COURSE CREDIT : 3 UNITS
PRE-REQUISITE : NONE

The course deals with the nature of mathematics, appreciation of its practical, intellectual
and aesthetic dimensions, and application of mathematical tools in daily life. The course begins
with an introduction to the nature of mathematics as an exploration of patterns (in nature and
environment) and as an application of inductive and deductive reasoning. By exploring these
topics, students are encouraged to go beyond the typical understanding of mathematics as
merely a set of formulas but as a source of aesthetics in patterns of nature, for example, a rich
language in itself (and of science) governed by logic and reasoning. The course proceeds to
survey ways in which mathematics provides a tool for understanding. And dealing with various
aspects of present day living, such as managing personal finances, making social choices,
appreciating geometric designs, understanding codes used in data transmission and security,
and dividing limited resources fairly. These aspects will provide opportunities for actually doing
mathematics in a broad range of exercises that bring out the various dimensions of mathematics
as a way of knowing and test the students’ understanding and capacity.

COURSE OBJECTIVES

After completing this course, you must be able to:

1. Discuss and argue about the nature of mathematics


2. Use a variety of statistical tools to process and manage numerical data
3. Appreciate the nature and uses of mathematics in everyday life.

COURSE REQUIREMENTS

1. A portfolio which consists of the all the answered activities and exercises will be
submitted at the end of the semester.
2. Other requirements such as written outputs, exercises, assignments and the likes will be
given throughout the sessions with set due dates by the teacher..
3. The course is expected to have a minimum of four (4) quizzes and two (2) major
examination (Midterm and Final Examination).
4. Students are encourage to attend the online class sessions to further enhanced their
understanding of the lesson.

Note: Some activities will be rated using the Rubrics.

iv
SUBJECT: GEED 10053: MATHEMATICS IN THE MODERN WORLD
PREPARED BY: Assoc. Prof. CHERRY E. ANGELES & WELFREDO B. QUEREMIT, LPT
GRADING SYSTEM
The grading system will determine if the student passed or failed the course. There will be two
grading periods: Midterm and Final Period. Each period has components of: 70% Class
Standing + 30% Major Examination. Final Grade will be the average of the two periodical
grades.

Midterm Finals
Class Standing 70% Class Standing 70%
 Quizzes  Quizzes
 Activities  Activities
 Portfolio  Project
Mid-term Examination 30%  Portfolio
Final Examination 30%
FINAL GRADE = (Midterm + Finals) /2

RUBRICS

Exemplary Satisfactory Developing Beginning Non-compliance


Criteria
1.00 - 1.25 1.50 – 1.75 2.00 - 2.50 2.75 - 3.50 4.00 - 5.00
The submitted The submitted The submitted The submitted No submitted
Assignment/ output manifests output manifests output partially output does not output
Activity qualities which go the required manifests the manifest any of
beyond the qualities required qualities. the requirements
requirements Certain aspects or certain aspects
are incomplete. are incorrect

COURSE GUIDE

Regular class (18 weeks, 3hrs/week, 54hrs)


Week Topic Learning Methodology Resources Assessment
Outcomes
Part 1: THE  Identify  Video Essential  Short-
NATURE OF patterns in watching Mathematics response
MATHEMATICS nature and for the Modern or essay
 Pair World by writing
regularities in
I. Mathematic the world. (K) sharing or Rizaldi Nocon
s in our small group and Ederlina  Two-to-
 Articulate the three page
Week World importance sharing Nocon
synthesis
1 to 2 1. Patterns of paper
and  Journal Nature’s
mathematics Numbers by Ian
numbers in writing
in one’s life. Stewart
Nature and
(V)  Class
the World
 Argue about Mathematics in
v
SUBJECT: GEED 10053: MATHEMATICS IN THE MODERN WORLD
PREPARED BY: Assoc. Prof. CHERRY E. ANGELES & WELFREDO B. QUEREMIT, LPT
2. The the nature of discussion Nature:
Fibonacci mathematics, Modelling
Sequence what it is , Patterns in the
3. Mathematic Natural World
how it is
s helps by John A.
expressed, Adam
organize represented
patterns and and used. Schaum’s
regularities (K) Outline:
in the world  Express Mathematics for
4. Mathematic the Liberal Arts
appreciation
s helps by Christopher
for
predict the Thomas (2009
mathematics McGraw-Hill
behavior or
nature and as a human Co.)
phenomena endeavor.
in the world. (V)
5. Mathematic
s helps
control
nature and
occurrences
in the world
for our own
ends.
6. Mathematic
s has
numerous
applications
in the world
making it
indispensabl
e.
 Discuss the  Individual Essential  Written
II. Mathematic language, or small Mathematics Exercise
al Language symbols, and group for the Modern set
and World by (seatwork)
conventions exercises
Week Symbols of including RizaldiNocon
3 1. Characteristi and  Quiz
mathematics. games
cs of (K) EderlinaNocon
mathematic  Explain the  Class
al language: discussion Learning the
nature of Language of
precise,
mathematics
concise,  Compilation Mathematics by
as a Jamison, R. E.
powerful of
language. (2000)
2. Expressions mathematic
vs. (K)
al symbols Language and
vi
SUBJECT: GEED 10053: MATHEMATICS IN THE MODERN WORLD
PREPARED BY: Assoc. Prof. CHERRY E. ANGELES & WELFREDO B. QUEREMIT, LPT
sentences  Perform and Learning
3. Conventions operations notations acroos
in the on Disciplines
mathematic mathematical
al language Schaums’
expressions Outline:
4. Four basic correctly. (S) Mathematics for
concepts:  Acknowledge the Liberal Arts
sets, that by Christopher
functions, Thomas (2009
mathematics
relations McGraw-Hill
is a useful
and binary Co.)
language.
operations
5. Elementary (V)
Logic:
connectives,
quantifiers,
negation,
variables
 Use different  Reading Essential  Take-
III. Problem types of and writing Mathematics home
Solving reasoning to proofs for the Modern problem
1. Inductive vs. World by set
justify
Week Deductive statements  Small- RizaldiNocon
4 Reasoning group and  Quiz on
and proving
2. Intuition, arguments problem EderlinaNocon
using
proof, and made about solving deductive
certainty Mathematical
mathematics. or
3. Polya’s 4-  Class Excursions by
(K) inductive
steps in discussions R. Aufmann
reasoning
 Write clear
Problem of key
and logical What is
Solving problems Mathematics
4. Problem proofs. (K)
and Really? by R.
Solving  Solve solutions Hersh
Strategies problems
5. Mathematic involving Schaums’
al Problems patterns and Outline:
involving recreational Mathematics for
Patterns problems the Liberal Arts
following by Christopher
Thomas (2009
Polya’s four
McGraw-Hill
steps. (S) Co.)
 Organize
one’s
methods and
approaches

vii
SUBJECT: GEED 10053: MATHEMATICS IN THE MODERN WORLD
PREPARED BY: Assoc. Prof. CHERRY E. ANGELES & WELFREDO B. QUEREMIT, LPT
for proving
and solving
problems.
(V)

Part 2:  Use a variety  Lectures Essential  Take-


MATHEMATICS of statistical Mathematics home
AS A TOOL tools to  Work with for the Modern problem
appropriate World by set
process and
Week I. Data manage statistical RizaldiNocon
5–8 Managem software and  Quiz
data. (S)
ent  Use methods EderlinaNocon
1. Data  Class
of linear
gathering discussion Mathematical
regression  Quantitativ
and Excursions by
and e
organizing  Pseudo- Aufmann
Study/Res
correlations proposal
data, data earch to
to predict the defense Mathematics: A
representa Practical
be orally
tion using value of a presented.
variable Odyssey by
graphs Johnson and
and given certain Mowry
charts; conditions.
interpretin (S) Schaums’
g  Advocate the Outline:
organized use of Mathematics for
data statistical the Liberal Arts
2. Measures by Christopher
data in
of central Thomas (2009
decision McGraw-Hill
tendency making. (V) Co.)
3. Measures
of
dispersion
4. Measures
of relative
position
5. Probabilite
s and
Normal
Distributio
ns
6. Linear
Regressio
n and
Correlation

Week Midterm

viii
SUBJECT: GEED 10053: MATHEMATICS IN THE MODERN WORLD
PREPARED BY: Assoc. Prof. CHERRY E. ANGELES & WELFREDO B. QUEREMIT, LPT
9 Examination

At this point, the instructor will choose 3 among the topics from the subsequent table.
 Use  Lectures Essential  Take-
Week II. Linear mathematical Mathematics home
s Programmi concepts and  Role- for the Modern problem
10 – ng playing World by set
tools in other
12 1. Linear areas.(S) RizaldiNocon
Inequalitie  Class and  Integrative
 Support the Project
s discussion EderlinaNocon
use of
2. Geometry mathematics
of Linear  Written
in various exercises
Programmi Mathematical
aspects and Excursions by
ng Models
endeavors. Aufmann
3. Simplex
Method (V)
Mathematics: A
Practical
Odyssey by
Johnson and
Mowry

Schaums’
Outline:
Mathematics for
the Liberal Arts
by Christopher
Thomas (2009
McGraw-Hill
Co.)

III. The  Use  Lectures Essential  Take-


Week Mathemati mathematical Mathematics home
s cs of concepts and  Role- for the Modern problem
13 – Finance playing World by set
tools in other
15 1. Simple areas.(S) RizaldiNocon
and  Class and  Integrative
 Support the Project
Compound discussion EderlinaNocon
use of
Interest mathematics
2. Credit  Written
in various exercises
Cards and Mathematical
aspects and Excursions by
Consumer
endeavors. Aufmann
Loans
3. Stocks, (V)
bonds, Mathematics: A
Practical
and Odyssey by
mutual Johnson and
funds
ix
SUBJECT: GEED 10053: MATHEMATICS IN THE MODERN WORLD
PREPARED BY: Assoc. Prof. CHERRY E. ANGELES & WELFREDO B. QUEREMIT, LPT
Mowry

Schaums’
Outline:
Mathematics for
the Liberal Arts
by Christopher
Thomas (2009
McGraw-Hill
Co.)

IV. Apportion  Use  Lectures Essential  Take-


Week ment and mathematical Mathematics home
s Voting concepts and  Role- for the Modern problem
16 – 1. Introductio playing World by set
tools in other
17 n to areas.(S) RizaldiNocon
apportion  Class and  Integrative
 Support the Project
ment discussion EderlinaNocon
use of
2. Introductio mathematics
n to Voting  Written
in various exercises
3. Weighted Mathematical
aspects and Excursions by
Voting
endeavors. Aufmann
Systems
(V)
Mathematics: A
Practical
Odyssey by
Johnson and
Mowry

Schaums’
Outline:
Mathematics for
the Liberal Arts
by Christopher
Thomas (2009
McGraw-Hill
Co.)

Week Final
18 Examination

x
SUBJECT: GEED 10053: MATHEMATICS IN THE MODERN WORLD
PREPARED BY: Assoc. Prof. CHERRY E. ANGELES & WELFREDO B. QUEREMIT, LPT
TABLE OF CONTENTS

Topic Page

Introduction ii

Part 1 The Nature of Mathematics

Lesson 1 Mathematics in our World 1

Unit 1 : Patterns and numbers in Nature and the World 1


Unit 2 : The Fibonacci Sequence 4
Unit 3 : Mathematics helps organize patterns and regularities
in the world 5
Unit 4 : Mathematics helps predict the behavior or nature and
phenomena in the world. 6
Unit 5 : Mathematics helps control nature and occurrences in
the world for our own ends. 7
Unit 6 : Mathematics has numerous applications in the world
making it indispensable 8

Lesson 2 Mathematical Language and Symbols 10

Unit 1 : Characteristics of mathematical language: precise,


concise, powerful 10
Unit 2 : Expressions vs. Sentences 11
Unit 3 : Conventions in the mathematical language 12
Unit 4 : Four basic concepts: sets, functions, relations and
binary operations 13
Unit 5 : Elementary Logic: connectives, quantifiers, negation,
variables 18

Lesson 3 Problem Solving 21

Unit 1 : Inductive vs. Deductive Reasoning 21


Unit 2 : Intuition, proof, and certainty 24
Unit 3 : Polya’s 4-steps in Problem Solving 27
Unit 4 : Problem Solving Strategies 29
Unit 5 : Mathematical Problems involving Patterns 32

Lesson 4 Data Management 36

Unit 1 : Data gathering and organizing data, data representation


using graphs and charts; interpreting organized data 36
Unit 2 : Measures of central tendency 43
Unit 3 : Measures of dispersion 47
Unit 4 :Measures of relative position 52
Unit 5 : Probability and Normal Distributions 55
Unit 6 : Linear Regression and Correlation 59

xi
SUBJECT: GEED 10053: MATHEMATICS IN THE MODERN WORLD
PREPARED BY: Assoc. Prof. CHERRY E. ANGELES & WELFREDO B. QUEREMIT, LPT
Part 2 Mathematics as a Tool

Lesson 5 Linear Programming 64

Unit 1 : Linear Inequalities 64


Unit 2 : Geometry of Linear Programming Models 66
Unit 3 : Simplex Method 69

Lesson 6 The Mathematics of Finance 78

Unit 1 : Simple and Compound Interest 78


Unit 2 : Credit Cards and Consumer Loans 81
Unit 3 : Stocks, bonds, and mutual funds 82

Lesson 7 Apportionment and Voting 85

Unit 1 : Introduction to apportionment 85


Unit 2 : Introduction to Voting 87
Unit 3 : Weighted Voting Systems 89

References 91

xii
SUBJECT: GEED 10053: MATHEMATICS IN THE MODERN WORLD
PREPARED BY: Assoc. Prof. CHERRY E. ANGELES & WELFREDO B. QUEREMIT, LPT
MATH 10053 - MATHEMATICS IN THE MODERN WORLD

PART 1 THE NATURE OF MATHEMATICS

LESSON 1 - MATHEMATICS IN OUR WORLD

Overview

This lesson will present the different aspects of appreciating nature in Mathematics in
the form of patterns. This will also allow students to grasp relationships between mathematics
and nature. .

Unit 1 - Patterns and Number in Nature and the World

Learning Objectives:

After completion of this lesson, you should be able to:

1. Identify patterns in nature and regularities in the world; and


2. Write a short essay about the nature of mathematics.
.
Course Materials:

In our everyday life, nature always surrounds us. We see beauty in nature! But have you
wonder what this beautiful surroundings mean to us? Or merely appreciate things happened
repeatedly?

Pattern means regular, repeated or recurring forms or designs. Patterns are everywhere.
We see pattern in plants such as the arrangements of stems, leaves and petals of flowers,
snowflakes, honeycombs, animal skin, snail’s shell, layout of tiles, designs of buildings and
many more. The following are some examples:

Source: Gilmour.com Source: patterninnature.org


The sunflower with pattern of clockwise and This starfish has a five-fold symmetry
counterclockwise spirals extending outward
from the center of the flower
SUBJECT: GEED 10053: MATHEMATICS IN THE MODERN WORLD 1
PREPARED BY: Assoc. Prof. CHERRY E. ANGELES & WELFREDO B. QUEREMIT, LPT
Pngwave.com wildfact.com
The cross-section of a nautilus mollusk shell Patterns are also featured in the skin or
shows a refined spiral structure called externals of animals, like reddish-orange fur
Equiangular spiral which states that as the and dark stripes of tigers. According to Alan
distance from the spiral center increases Turing, chemical reactions and diffusion
(radius),the amplitude of the angles formed processes in cells determine these growth
by the radii to the point and the tangent to patterns.
the point remain constant.

Read:
https://astrofella.wordpress.com/2019/03/31/natures-numbers-ian-stewart/

Watch:
https://vimeo.com/9953368

https://etereaestudios.com/works/infinitepatterns/

Activities/Assessments:

1. What is the next figure in the pattern below?

______________ ?

A B
2. Which figure logically belongs on the spot of the question mark?

2
SUBJECT: GEED 10053: MATHEMATICS IN THE MODERN WORLD
PREPARED BY: Assoc. Prof. CHERRY E. ANGELES & WELFREDO B. QUEREMIT, LPT
a.. b. c. d. e.

3. Determine what comes next in the given patterns.


a. A, C, E, G, I, _____
b. 15, 10, 14, 10, 13, 10, _____
c. 3, 6, 12, 24, 48, 96, _____
d. 27, 30, 33, 36, 39, _____
e. 41, 39, 37, 35, 33, ______
4. Write the next three terms of the following

a. -4, -1, 2, ___, ____, ____.

b. 5, 9, 13, ___, ____, ____ .

c. 6, 9, 14, 21, ___, ___, ____.

d. 1/2, 2/3, 3/4, ___, ___, ____.

e. 7, 17, 27, 37, ___, ___, ____.

5.. Look for the meaning of the following terms and give example(s)

a. Symmetry b. Order of Rotation (given the n-fold rotational symmetry) c. Packing Problems

6. Give other examples of patterns in nature. Discuss.

7. Write a short essay/article about the Nature of Mathematics.

References:

Mathematics in the Modern World by Aufmann, Richard, et. al.


Natures Numbers by Ian Stewart
https://www.youtube.com/watch?v=WxeHtY-H7Dk

3
SUBJECT: GEED 10053: MATHEMATICS IN THE MODERN WORLD
PREPARED BY: Assoc. Prof. CHERRY E. ANGELES & WELFREDO B. QUEREMIT, LPT
https://www.khanacademy.org/math/cc-third-grade-math/arithmetic-patterns-and-
problem-solving/imp-patterns-in-arithmetic/v/practice-finding-patterns-in-numbers

Unit 2 - The Fibonacci Sequence

Learning Objectives:

After completion of this lesson, you should be able to:

1. Enumerate the Fibonacci sequence; and


2. Compute and solve problems involving Fibonacci sequence
Course Materials:

A sequence is an ordered list of numbers, called terms, that may have repeated
values. The arrangement of these terms is set by a definite rule. The Fibonacci sequence
exhibits a certain numerical pattern which originated as the answer to an exercise in the first
ever high school algebra text. This pattern turned out to have an interest and importance far
beyond what its creator imagined. It can be used to model or describe an amazing variety of
phenomena, in mathematics and science, art and nature. The mathematical ideas the Fibonacci
sequence leads to, such as the golden ratio, spirals and self- similar curves, have long been
appreciated for their charm and beauty, but no one can really explain why they are echoed so
clearly in the world of art and nature.

Source: https://math.temple.edu/~reich/Fib/fibo.html

Symbolically,

fn = number of pairs during month n

fn = fn-1 + fn-2

4
SUBJECT: GEED 10053: MATHEMATICS IN THE MODERN WORLD
PREPARED BY: Assoc. Prof. CHERRY E. ANGELES & WELFREDO B. QUEREMIT, LPT
So, the number sequence 1,1,2,3,5,8,13,21,33, … composes the Fibonacci Sequence.

Watch:
https://www.youtube.com/watch?v=dREpRHgkjsg
https://www.youtube.com/watch?v=RjM8AaNSjhA (2:39
https://www.youtube.com/watch?v=nt2OlMAJj6o

Activities/Assessments:

1. Let Fib(n) be the nth term of the Fibonacci sequence with Fib(1)=1, Fib(2)=1, Fib(3)=2, and
so on. Find:
a. Fib(8)
b. Fib(19)
c. Fib(23) if Fib(22)=17,711 and Fib(24)=46,368
2. What will be the sum of Fib(1) + Fib(2) + …+ Fib(10)? Determine the pattern in the
successive sums from the previous question.

3. Art Activities:

Fibonacci Sequence follows the Golden Ratio (Φ). Each succeeding terms
represents the dimensions of successive rectangles. Using a graphing
paper,draw a Fibonacci Spiral whose Fib(1) = 3 upto Fib(5).

3. Show that each terms of the Fibonacci Sequence: 1, 1, 2, 3, 5, 8, shows a Golden Ratio (Φ) ≈
1.618

References:

Mathematics in the Modern World by Aufmann, Richard, et. al.

Unit 3 - Mathematics helps organized patterns and regularities in the world

Learning Objectives:

At the end of this lesson, you should be able to:

1. articulate the importance of Mathematics in one’s life; and


2. Give examples of different forms of nature patterns.

Course Content:

In this world, patterns and regularities are always present. We may not notice that things
in our environment contributes a lot in the concept of mathematical modelling. Natural patterns
maybe considered as symmetries, fractals, tessellation, stripes, spirals, cracks and foam.

Examples are the following:


5
SUBJECT: GEED 10053: MATHEMATICS IN THE MODERN WORLD
PREPARED BY: Assoc. Prof. CHERRY E. ANGELES & WELFREDO B. QUEREMIT, LPT
1. Symmetry - when different sides of something are alike. Ex. Butterflies,face, starfish
2. Fractals - a detailed pattern that look similar at any scale and repeats itself overtime.
Ex. Cracked in ice, lighting bolt, fern.
3. Tessellations - are patterns that are formed by repeated cubes or tiles.
Ex. Pineapple, honeycomb, turtle.
4. Other nature patterns are spirals (Rose), stripes (coconut and palm leaves), cracks
(cracked land or walls) and foam (soap foam bubbles).

Watch:
https://www.youtube.com/watch?v=dLU78Xyb0io
Read:
https://mathematicsinourlife.wordpress.com/

Activities/Assessment:

1. Clip a picture of other examples other forms of patterns and regularities in nature.
2. In 150 words, discuss how mathematics helps organized patterns and regularities in
nature.

References:
Mathematics in the Modern World by Aufmann, Richard, et. al.
Wordpress.com

Unit 4 - Mathematics helps predict the Behaviour or Nature and Phenomena in the
World

Learning Objectives:

At the end of this lesson, you should be able to:

1. Argue about the nature of mathematics, what is, how it is expressed and used;
2. Express appreciation for mathematics as a human endeavor; and
3. Solve population growth.

Course Materials:

Mathematics can also be used to model population growth. The formula for exponential
growth is A = Pert where A is the size of the population after it grows, P is the initial number of
people, r is the rate of growth and t is time. Racall that e is Euler’s constant with an approximate
value of 2.718.

6
SUBJECT: GEED 10053: MATHEMATICS IN THE MODERN WORLD
PREPARED BY: Assoc. Prof. CHERRY E. ANGELES & WELFREDO B. QUEREMIT, LPT
Example. The exponential growth model A = 30e0.02t describes the population of a town
in Cavite in thousands, t years after 2010. What will be the population in 2025?

Solution:

Given: P = 30, t = 15

So, using the formula,

A = 30e0.02(15)
= 30 (1.34985…)
= 40.4957

Therefore, the town population would be about 40,496 in 2025.

Activities/Assessment:

1. Submit an example of weather forecast for 1 week in your location. Compute for the
average temperature in a week span.
2. Solve: The exponential growth model A = 50e0.07t describes a population of a city in
the Philippines in thousands, t years after 2010. What is the population after 20 years?

References:
Mathematics in the Modern World by Aufmann, Richard, et. al.

Unit 5 - Mathematics helps control nature and occurrences in the world for our
own ends.

Learning Objectives:

At the end of this lesson, you should be able to:

1. Discuss the importance of mathematics in controlling nature and occurrences.

Course Materials:

In history, astronomers and scientists were able to discover and formulate things that
helps us to ;live in harmony with nature. From the time we woke up in the morning, as the sun
shines up to sunset until we close our eyes to sleep, mathematics helps us do this with
prediction. We can calculate how many hours will be spent for work, for family and leisure
because of day and night schedules. Even weather can be predicted as if it will be sunny or
rainy, or will there be a storm coming. Scientists can do weather forecasting because of
historical patterns while astronomers can also use patterns to predict occurrences such as
meteor showers and eclipses.

7
SUBJECT: GEED 10053: MATHEMATICS IN THE MODERN WORLD
PREPARED BY: Assoc. Prof. CHERRY E. ANGELES & WELFREDO B. QUEREMIT, LPT
Watch:
https://www.worldsciencefestival.com/videos/go-figure-predicting-the-
world-with-math/

Read:
https://www.academia.edu/37079573/MATHEMATICS_HELPS_PREDICT_THE_BEHAVIOR
_OF_NATURE_AND_PHENOMENA_IN_THE_WORLD

Activities/Assessment:

1. Why mathematics helps control nature and occurrences in the world for our own ends?
Explain.

References:
Mathematics in the Modern World by Aufmann, Richard, et. al.

Unit 6 - Mathematics has numerous applications in the world making it


indispensable

Learning Objectives:

At the end of this lesson, you should be able to:

2. Explain why mathematics is indispensable.

Course Materials:

How is it possible that mathematics, a product of human thought that is independent of


experience, fits so excellently in the objects of reality?
- Albert Einstein
Mathematics is part of our everyday life. We can do all things because of Math. We can
calculate the energy used everyday in electricity, gasoline consumption, calories we eat, energy
used in doing work, etc. Our lives are a lot more easier with Mathematics behind. We can cook
with correct measurement of ingredients, we can invest in stock markets, house and lot, and few
properties that we know can give better profits. We can think critically about making decisions
and solve problems analytically.

According to this line of argument, reference to (or quantification


over) mathematical entities such as sets, numbers, functions and such is indispensable to our
best scientific theories, and so we ought to be committed to the existence of
these mathematical entities. (Source: https://plato.stanford.edu/archives/spr2019/entries/mathphil-indis/)

8
SUBJECT: GEED 10053: MATHEMATICS IN THE MODERN WORLD
PREPARED BY: Assoc. Prof. CHERRY E. ANGELES & WELFREDO B. QUEREMIT, LPT
Activities/Assessment:

3. Make a short discussion: Why mathematics indispensable?

References:
Mathematics in the Modern World by Aufmann, Richard, et. al.

9
SUBJECT: GEED 10053: MATHEMATICS IN THE MODERN WORLD
PREPARED BY: Assoc. Prof. CHERRY E. ANGELES & WELFREDO B. QUEREMIT, LPT
LESSON 2: Mathematical Language and Symbols
Overview

This lesson introduces foundation for mathematical thought, mathematical language,


sets, functions, relations and mathematical logic.

Learning Objectives:

After completion of this lesson, you should be able to:

1. Discuss the language, symbols and conventions of mathematics;


2. Translate sentences into mathematical statements;
3. Perform operations on mathematical expressions correctly.

Course Materials:

Unit 1 - Characteristics of Mathematical language

Many students sometimes make it hard to understand mathematical ideas, not because
they are difficult, but they are presented using foreign language, the language of mathematics.

The following are the characteristics of mathematical language:


1. Precise - able to find very fine distinction
2. Concise - able to say things briefly
3. Powerful - able to express complex thoughts with ease.

Read:
http://www.onemathematicalcat.org/pdf_files/LANG1.pdf

Activities/Assessments:

1. Prepare a compilation of mathematical symbols and notations and their meaning.


2. In a whole sheet of yellow pad, write an essay about the “Importance of Mathematical
Language in Understanding Mathematics” (minimum of 150 words and three paragraphs).

References:

https://www.researchgate.net/publication/315712910_The_Symbolic_Language_of_Mat
hematics#:~:text=Mathematics%20is%20written%20in%20a,facts%20are%20expressed
%20in%20symbolism.

Unit 2 - Expressions vs. Sentences

Mathematical expressions consist of letters and numbers or combination of both.


10
SUBJECT: GEED 10053: MATHEMATICS IN THE MODERN WORLD
PREPARED BY: Assoc. Prof. CHERRY E. ANGELES & WELFREDO B. QUEREMIT, LPT
Examples:

1. 2a2 , a + b, m, a3b5 + c.

2. Translate the following phrases into expressions

a. The sum of two numbers multiplied by seven

Answer: If a and b are the two numbers then the expression is 7(a + b)

b. The square root of the difference of two numbers.

Answer: If a and b are the two numbers then the expression is √(a – b)

Mathematical statements make use of at least two expressions. They also use
mathematical symbols such as equals, greater than, less than.Examples are a + 2b2 = 10,
1 +5 > 2a, 3 -2 = 1/2c.

Examples: Translate the following sentences into equations

1. The sum of two numbers multiplied by 7 is 21

Answer: If a and b are the two numbers then the equation is 7(a + b) = 21

2. The square root of the difference of two numbers is 4

Answer: If a and b are the two numbers then the equation is √(a – b) = 4

Activities/Assessments:

Translate the following:


A. Phrases into expressions
1. Twice a number diminished by seven
2. Six more than thrice a number
3. Seven times a number less than four times of another number
4. Half of the quotient of two numbers
5. Seven subtracted from the product of two numbers.

B. Sentences into equations


1. Twice a number diminished by seven is three.
2. Six more than thrice a number is ten.
3. Seven times a number less than four times of another number is twenty-five.
4. Half of the quotient of two numbers is eight.
5. Seven subtracted from the product of two numbers is thirteen.

11
SUBJECT: GEED 10053: MATHEMATICS IN THE MODERN WORLD
PREPARED BY: Assoc. Prof. CHERRY E. ANGELES & WELFREDO B. QUEREMIT, LPT
C. Expressions into phrases
1. 5x – 7
2. (a + b) ÷ 8
3. a2 + b2 – c2
4. ½ m + ½ n
5. 6(p + q)

D. Equations into sentences


1. 5x – 7 = 3
2. (a + b) ÷ 8 = 2
3. a2 + b2 = c2
4. ½ m + ½ n = 2(m-n)
5. 6(p + q) = 12 + 6p

References:

http://www.onemathematicalcat.org/pdf_files/LANG1.pdf

Unit 3 - Conventions in the mathematical language

A mathematical convention is a fact, name, notation, or usage which is generally


agreed upon by mathematicians. For instance, the fact that one evaluates multiplication before
addition in the expression is merely conventional: there is nothing inherently
significant about the order of operations. Mathematicians abide by conventions in order to allow
other mathematicians to understand what they write without constantly having to redefine basic
terms. (Imagine if every mathematical paper began with an explanation of PEMDAS!)

Nearly all mathematical names and symbols are conventional. The longer a name or notation
has been in use, the more likely it is to become a mathematical convention. Unfortunately, some
notational questions stubbornly refuse to develop conventional solutions, usually because two
or more competing conventions achieve wide-spread usage. See, for example, the article on the
Natural numbers.

Alternate Meaning

In English, a convention is also "a place where people convene, or come together." Thus,
the phrase "mathematical convention" is also used to denote a convention whose purpose is
mathematical. For instance, Mu Alpha Theta describes its yearly gatherings as conventions.

Variables are letters that can assume many values, I.e. a,b,c,A,B,C,… Variables help
translate sentences into mathematical statement.

Example: Use variables to rewrite the following sentences mathematically.

1. Are there numbers with the property that the sum of their squares equals the square
of their sum?
12
SUBJECT: GEED 10053: MATHEMATICS IN THE MODERN WORLD
PREPARED BY: Assoc. Prof. CHERRY E. ANGELES & WELFREDO B. QUEREMIT, LPT
Answer: Are there numbers a and b with the property that a2 + b2 = (a + b)2?
Or: Are there numbers a and b such that a2 + b2 = (a + b)2?
Or: Do there exist any numbers a and b such that a2 + b2 = (a + b)2?

2. Given any real number, its square is non negative.

Answer: Given any real number r, r2 is non negative.


Or: For any real number r, r2 ≥ 0.
Or: For all real number r, r2 ≥ 0.

Activities/Assessments:

In a yellow pad paper, make a reflection about the Conventions in the Mathematical
Language. (three paragraphs with minimum of 100 words)

Assignment

Make a comic strip about “The World where Mathematics is a Language”

References:

http://www.onemathematicalcat.org/pdf_files/LANG1.pdf

Unit 4 - Four Basic Concepts

Sets
Sets as defined by George Cantor (1845-1918) it is simply a collection of objects or
elements with a common description as set of fruits, set of people living in a city, set of numbers,
set of odd numbers, and many more.

If an object belongs to the set, it is called a member or element of the set denoted by
the symbol “∈“. If the object does not belong to the set, it is not the element of the set denoted
by the symbol “∉” It is usually written by enclosing elements with braces, separated by comma.
Capital letter is used to denote a set. A subset is a part of set. If A and B are sets, then A is a
subset of B (A  B) iff every element of A is also an element of B. Set can be described
mathematically by using the Roster Method.

A set is said to be well-defined if we know exactly if an object is an element of the set.

Examples:

A = {a, e, I, o, u} – set A is the set of vowels in the English Alphabet

a ∈ A, read as “a is an element of set A”


13
SUBJECT: GEED 10053: MATHEMATICS IN THE MODERN WORLD
PREPARED BY: Assoc. Prof. CHERRY E. ANGELES & WELFREDO B. QUEREMIT, LPT
b ∉ A, read as “b is not an element of set A”

Set A is a well-defined set.

Equal and Equivalent Sets

Two sets are equal of they have the same elements

Examples:
Set A = {4, 1, 3} Set B = {3, 1, 4} Set C = {3, 1, 5}

Set A and Set B are equal sets or A = B because they have the same elements.
Set A and Set Care not equal sets or A ≠ C because their elements are not exactly the
same.

Two sets are equivalent if they have the same number of elements

Examples:
Set D = {1, 2, 3} Set F = {g, e, f} Set E = {1, 2, 3, 4}

Set D is equivalent to Set F because they have the same number of elements.
Set D and Set E are not equivalent because they do not have the same number of
elements.

Universal Sets and Subsets

A universal set is a set that contains of all the elements being considered in a particular
problem. The universal set is represented by the letter U.

Examples:
a. Suppose we are considering whole numbers, then:
U = {0, 1, 2, 3, 4,…}

b. If Set A = {1, 2, 3, 4},Set B = {5, 6, 7, 8, 10}, then:


U = {1, 2, 3, 4, 5, 6, 7, 8, 10}

Set A is said to be a subset of set B if every element of A is also an element


of B. We symbolize this concept by A ⊆ B read as “A is a subset of B” or “A contained in
B”

Examples:

14
SUBJECT: GEED 10053: MATHEMATICS IN THE MODERN WORLD
PREPARED BY: Assoc. Prof. CHERRY E. ANGELES & WELFREDO B. QUEREMIT, LPT
1. Let A = {4, 5, 6} and B ={1, 2, 3, 4, 5, 6, 7, 8} then A is a subset of B because all the
elements of A also contained in B. A ⊆ B
2. Let A = {1, 2, 3} find all the possible subsets of Set A
Subsets are: {1}, {2}, {3}, {1, 2}, {2, 3}, {1, 3}, { }
Note: Empty set is a subset of any set with at least one element.

Intersection and Union of Sets

The intersection of two sets A and B denoted by A ∩ B is the set of all elements
which are common to both A and B.

The union of two sets A and B denoted by A ∪ B is the set of all elements which
belong to A or to B or to both A and B.

Venn Diagram is used to represent the relationships between two or more sets.

EXAMPLES

Given: U = {1, 2, 3, 4, 5, 6, 7, 8, 9, 10, 11, 12}, A = {3, 6, 9, 12} and B = {2, 4, 6, 8}


a. Find A ∩ B, Answer: A ∩ B = {6}
b. Find A ∪ B, Answer: A ∪ B = {2, 3, 4, 6, 8, 9, 12}
c. Draw a Venn Diagram to show the intersection
d. Draw a Venn Diagram to show the union

c. d.
U 7 U 7
A B 10 A B 10

1 3 9 161 2 4 1 3 9 6 2 4
12 8 12 8
5 11 5 11

Intersection Union

Complement of a Set and Difference of Two Sets

The complement of a set A denoted by Ac or A’ is the set of all elements in the universal
set that are not in A.

The difference of sets A and B denoted by A – B is the set of all elements which belong
to A but which do not belong to B.

EXAMPLES;
15
SUBJECT: GEED 10053: MATHEMATICS IN THE MODERN WORLD
PREPARED BY: Assoc. Prof. CHERRY E. ANGELES & WELFREDO B. QUEREMIT, LPT
Given: U = {1, 2, 3, 4, 5, 6, 7, 8, 9, 10, 11, 12}, A = {3, 6, 9, 12} and B = {2, 4, 6, 8}
a. Find A’, Answer: A’ = {1, 2, 4, 5, 7, 8, 10, 11}
b. Find A – B and B – A, Answer: A – B = {3, 9, 12} and B – A = {2, 4, 8}
c. Draw a Venn Diagram to illustrate A’
d. Draw a Venn Diagram illustrating A – B and B – A.

c. d.
U 7 U 7
A B 10 A B 10

1 3 9 161 2 4 1 3 9 6 2 4
12 8 12 8
5 11 5 11

Complement of Set A Difference of Sets A – B

d. .
U 7
A B 10

1 3 9 161 2 4
12 8
5 11
Difference of Sets B – A
Cartesian Product

Given sets A and B, the Cartesian product of A and B denoted by A x B is the set of all
ordered pairs (a, b), where a is in A and b is in B. In symbols,

A x B = {(a, b) / a ϵ A and b ϵ B}.

Example:
Let A ={1, 2, 3} and B = { m, n}. Find A x B.

Solution:
A x B = {(1, m), (1, n), (2, m), (2,n), (3, m), (3, n)}

Relations

Let A and B be sets. A relation R from A to B is a subset of A x B. Given an ordered pair


(x, y) in A x B, x is related to y by R, written x R y, iff (x, y) is in R. The set A is called the domain
of R and the set B is called its co-domain.

16
SUBJECT: GEED 10053: MATHEMATICS IN THE MODERN WORLD
PREPARED BY: Assoc. Prof. CHERRY E. ANGELES & WELFREDO B. QUEREMIT, LPT
Functions
A function F from a set A to a set B is a relation with domain A and co-domain B that
satisfies the following properties:
1. For every element of A is the first element of an ordered pair of F.
2. No two distinct ordered pairs in F have the same first element.

Examples:
Let A ={ 2,4,6} and B = { 1, 3, 5}.Which of the following relations R, S and T defined
below are functions from A to B?

1. R = {(2, 5), (4, 1), (4, 3), (5, 5)}


2. For all (x, y) ϵ A x B, (x, y) ϵ S means that y = x + 1.
3. T is defined by the arrow diagram

2 1
4 3
6 5

Solution:
1. R is not a function since two ordered pairs (4,1) and (4, 3_ have the same first
element.
2. S is not a function because it does not satisfy property (1). For ex. 6 is an element of
A but there is no y in B such that y = 6 + 1 = 7.
3. T is a function, Each element of A has a corresponding element in B and there is no
element of the domain that has more than one arrow coming out of A.

Activities/Assessments:

1. Given: U = {1, 2, 3, 4, 5, 6, 7, 8, 9, 10}, A = {1, 3, 5, 7, 9}, B = {2, 4, 6, 8 10}

C = {2, 4, 8} and D = {3, 6, 9}

Find: 1. A ⊆ D 6. A ∩ C 11. B ∪ C
2. A – D 7. C ∪ D 12. B’
3. A’ 8. C’ 13. D – A
4. C – B 9. A – C 14. C – D

5. C ⊆ B 10. B ∩ C 15. A ∪ B

2. Draw Venn Diagrams illustrating items 11 - 15

17
SUBJECT: GEED 10053: MATHEMATICS IN THE MODERN WORLD
PREPARED BY: Assoc. Prof. CHERRY E. ANGELES & WELFREDO B. QUEREMIT, LPT
References:

http://www.onemathematicalcat.org/pdf_files/LANG1.pdf

Unit 5 - Elementary Logic

Logic Statements
A statement is a declarative sentence that is either true or false, but not both true and
false. Simple statements is a statement that conveys single idea while a compound
statement is a statement that conveys two or more ideas. Connecting simple statements with
and, or, if … then…, and if and only if creates a compound statement.

George Boole used symbols such as p, q, r and s to represent simple statements and
to represent connectives.

Truth Value and Truth Tables

The truth value of a simple statement either true (T) or false (F).

The truth value of a compound statement depends on the truth value of its simple

18
SUBJECT: GEED 10053: MATHEMATICS IN THE MODERN WORLD
PREPARED BY: Assoc. Prof. CHERRY E. ANGELES & WELFREDO B. QUEREMIT, LPT
statements and connectives.

A truth table is a table that shows the truth value of a compound statement for all
Possible truth values of its simple statements.

The negation of the statement “Today is Friday”. is the statement “Today is not
Friday”. In symbolic logic, the tilde ~ is used to denote negation of a statement.

EXAMPLES

1. Write the negation of each statement:


a. Ellie Goulding is an open opera.
b. The dog does not need to be fed.

Solution:
a. Ellie Goulding is not an opera.
b. The dog needs to be fed.

2. Consider the following simple statements.


p: Today is Friday.
q: It is raining .
r: I am going to a movie.
s: I am not going to the basketball game.

Write the following compound statements in symbolic form


a. Today is Friday and it is raining.
b. It is not raining and Ia am going to a movie.
c. I am going to the basketball game or I am going to a movie.
d. If it is raining, then I am not going to the basketball game.
Solution:
a. p ∧,q b. ~q ∧ r c. ~s ∨ r d. q → s

Quantifiers and Negation

In a statement, the word some and the phrases there exists and at least one are called
existential quantifiers and used as prefixes to assert the existence of something. The words
none, no, all, and every are called universal quantifiers.

Quantified Statements and Their Negation


Statement Negation
All X are Y Some X are not Y
No X are Y Some X are Y
Some X are not Y All X are Y
Some X are Y No X are Y

19
SUBJECT: GEED 10053: MATHEMATICS IN THE MODERN WORLD
PREPARED BY: Assoc. Prof. CHERRY E. ANGELES & WELFREDO B. QUEREMIT, LPT
Watch:
https://www.youtube.com/watch?v=0Zp0o8z125w

Activities/Assessments:

1. Explain the relationship of sets, relations and functions.


2. Write each symbolic statements in words. Use p, q, r, s and t as defined:
p: The tour goes to Italy.
q: The tour goes to Spain.
r: We go to Venice.
s: The meals are not included.
t: The hotel fees are included.
a. p ˄ ~q
b. r s
c. p q
d. p r
e. p t
References:
Mathematics in the Modern World by Aufmann, Richard, et. al.
https://www.researchgate.net/publication/315712910_The_Symbolic_Language_of_Mat
hematics#:~:text=Mathematics%20is%20written%20in%20a,facts%20are%20expressed
%20in%20symbolism.

http://www-math.ucdenver.edu/~wcherowi/courses/m3000/lecture2.pdf

20
SUBJECT: GEED 10053: MATHEMATICS IN THE MODERN WORLD
PREPARED BY: Assoc. Prof. CHERRY E. ANGELES & WELFREDO B. QUEREMIT, LPT
LESSON 3: Problem Solving

Overview

This lesson discusses the different problem solving methods that will help you acquire
problem solving skills and shows that problem solving can be an enjoyable journey.

Learning Objectives:

After completion of the lesson, you should be able to:

1. Use different types of reasoning to justify statements and arguments made about
mathematics;
2. Write clear and logical proofs;
3. Solve problems involving patterns and recreational problems involving Polya’s four
steps; and
4. Organize one’s methods and approaches for proving and solving problems.

Course Materials:

Unit 1 - Inductive and Deductive Reasoning

Reasoning and Logic


First, let's discuss the concept of reasoning. Reasoning is the action of constructing
thoughts into a valid argument. This is something you probably do every day. When you make a
decision, you are using reasoning, taking different thoughts and making those thoughts into
reasons why you should go with one option over the other options available. When you
construct an argument, that argument will be either valid or invalid. A valid argument is
reasoning that is comprehensive on the foundation of logic or fact.
Now let's discuss propositional logic. Inductive and deductive reasoning are both forms
of propositional logic. Propositional logic is the branch of logic that studies ways of joining
and/or modifying entire propositions, statements or sentences to form more complicated
propositions, statements or sentences. For our purposes, this means that propositional logic
uses a series of facts and reasoning to develop a conclusion. Inductive and deductive reasoning
use propositional logic to develop valid arguments based on fact and reasoning. Both types of
reasoning have a premise and a conclusion. How each type of reasoning gets to the conclusion
is different.

Inductive Reasoning is the process of reaching a general conclusion by examining specific


examples.
Uses:
1. To predict a number.
2. To make a conjecture
3. To solve an application
4. To make counterexample

21
SUBJECT: GEED 10053: MATHEMATICS IN THE MODERN WORLD
PREPARED BY: Assoc. Prof. CHERRY E. ANGELES & WELFREDO B. QUEREMIT, LPT
EXAMPLES
1. Use inductive reasoning to predict the next number in each of the following lists.
a. 3, 6, 9, 12, 15, ? b. 1, 3, 6, 10, 15, ?
SOLUTION
a. Each successive number is 3 larger than the preceding number. Thus the next
number in the list is 3 larger than 15, which is 18.
b. The first two numbers differ by 2. The second and the third numbers are differ by 3. It
appears that the difference between any two numbers is always 1 more than the
preceding difference. Since 10 and 15 is differ by 5, we predict that the next number
in the list will be 6 larger than 15, which is 21.
2. Use inductive reasoning to make a conjecture
Consider the following procedure: Pick a number. Multiply the number by 8, add 6 to the
product, divide the sum by 2, and subtract 3.
Complete the above procedure for several different numbers. Use inductive
reasoning to make a conjecture about the relationship between the size of the resulting
number and the size of original number.
SOLUTION
Suppose we pick 5 as our original number. Then the procedure would produce
the following results:
Original number: 5 Original number: 6
Multiply by 8: 8 x 5 = 40 Multiply by 8: 8 x 6 = 48
Add by 6: 40 + 6 = 46 Add by 6: 48 + 6 = 54
Divide by 2: 46 ÷ 2 = 23 Divide by 2: 54 ÷ 2 = 27
Subtract by 3: 23 – 3 = 20 Subtract by 3: 27 – 3 = 24
And so on … In each of these cases the resulting number is four times the
original number. We conjecture that the following the given procedure produces a
number that is four times the original number.

Deductive Reasoning is the process of reaching a conclusion by applying general principles


and procedures.

EXAMPLES

Use deductive reasoning to show that the following procedure produces a number that is
four times the original number.

Procedure: pick a number. Multiply the number by 8, add 6 to the product, divide
the sum by 2, and subtract 3.

22
SUBJECT: GEED 10053: MATHEMATICS IN THE MODERN WORLD
PREPARED BY: Assoc. Prof. CHERRY E. ANGELES & WELFREDO B. QUEREMIT, LPT
SOLUTION
Let n represent the original number.
Multiply the number by 8: 8n
Add 6 to the product: 8n + 6
Divide the sum by 2: (8n + 6) ÷ 2 = 4n + 3
Subtract 3: 4n + 3 – 3 = 4n
We started with n and ended with 4n. The procedure given in this example produces a
number that is four times the original number.
Inductive vs. Deductive Reasoning

Example:

Determine whether the following arguments is an example of inductive or deductive reasoning:


1. During the past ten years , a tree has produced plums every other year. Last year the
tree did not produce plums, so this year the tree will produce plums.

2. All home improvements cost more than the estimate.The contractor estimated that my
home improvement will cost $35,000. Thus, my home improvement will cost more than
$35,000.
Solution:
1. This is an example of inductive reasoning because the argument reaches a conclusion
based on specific examples.

2. This is an example of deductive reasoning because the conclusion is a specific case


of general assumption.

Activities/Assessments:

A. Determine whether each of the following arguments is an example of inductive


reasoning or deductive reasoning.
1. Amanda was learning to drive when her father told her to never run the car if the
overheat light comes on. Last summer while driving to Anchorage, her overheat light
turned on so she turned off the car and called up her.
2. Your teacher tells the class that no credit will be given to assignments that show no
work. For two months you do your homework. Last week, your assignment had a couple
of problems where no work shown. You received no credit for the assignment.
3. Hunter notices that every day his math teacher wears what seems to be different tie
every day. He starts keeping track of what his teacher wears. After one semester,
Hunter approaches his teacher and states, “I think you wear a different tie every day, do
you have a 180 different ties?”
4. All Gilliian Flynn novels are worth reading. The novel Gone Girl is a Gillian Flynn novel.
Thus Gone Girl is worth reading.

23
SUBJECT: GEED 10053: MATHEMATICS IN THE MODERN WORLD
PREPARED BY: Assoc. Prof. CHERRY E. ANGELES & WELFREDO B. QUEREMIT, LPT
5. I know I will win a jackpot on this slot machine in the next tries, because it has not
paid out any money during the last 45 tries.
6. Two computer programs, a bubble sort and a shell sort data. In each of 50
experiments, the shell sort program took less time sort the data than did the bubble sort
program. Thus the shell sort program is the faster of the two sorting programs.

Assignment:

A. Inductive Reasoning
1. Use inductive reasoning to predict the next number in each of the following lists.
a. 5, 10, 15, 20, 25, ? b. 2, 5, 10, 17, 26, ?
2. Consider the following procedure: Pick a number. Multiply the number by 9, add 15 to
the product, divide by the sum by 3, and subtract 5.
B. Deductive Reasoning
1. Procedure: Pick a number. Multiply the number by 6, add 10 to the product, divide the
sum by 2, and subtract 5. (let n be the original number).
2.Use deductive reasoning to determine the missing numbers in the magic squares.

2 13

10 11

6 12

4 15 1

References:
http://www-math.ucdenver.edu/~wcherowi/courses/m3000/lecture2.pdf

Unit 2 - Intuition Proof and Certainty

In the philosophy if mathematics, intuitionism or neointuitionism is an approach where


mathematics is considered purely the result of constructive mental ability of humans rather than
discovery of fundamental principles claimed to exist in the objective reality.

A proof is an inferential argument for mathematical statement. In the argument, other


previously established statements such as theorems can be used. In principle, a proof can be
traced back to self-evident or assumed statements known as axioms along with accepted rules
of inference.

24
SUBJECT: GEED 10053: MATHEMATICS IN THE MODERN WORLD
PREPARED BY: Assoc. Prof. CHERRY E. ANGELES & WELFREDO B. QUEREMIT, LPT
Three Basic Proof Techniques Used in Mathematics

1. Direct Proof
2. Proof by Contradiction
3. Proof by Induction

EXAMPLES
Prove that P → Q using the three techniques.
In each case, we will prove the following statmente:
1. The sum of any two consecutive numbers is odd.
SOLUTION

To do so, we will use the following:

Definition 1: An integer number ‘n’ is even if and only if there exists an integer ‘k’
such that n = 2k

Definition 2: An integer number ‘n’ is odd if and only of there exists an integer ‘k’
such that n = 2k + 1

Definition 3: Two integers ‘a’ and ‘b’ are consecutive if and only if b = a + 1

THE STATEMENT: The sum of any two consecutive numbers is odd may
not feel like P → Q statement, but of course we can phrase it:

If ‘a’ and ‘b’ are consecutive numbers, then of ‘a’ and ‘b’ is odd

DIRECT PROOF:

The basic idea of a direct proof of P → Q is: Assume that P is true. Use P
to show that Q is true.

Proof: Assume ‘a’ and ‘b’ are consecutive


Know that b = a + 1 so… a + b = a + a + 1 = 2a + 1 → 2k + 1 where k is
an integer.

PROOF BY CONTRADICTION

The idea here is that a proposition is either true or false, but not both. We
get a contradiction when we can show a statement is both true and false,
showing our initial assumptions are inconsistent. We use this to show P → Q by
assuming both P and not Q are simultaneously true. Deriving a contradiction.
Assume: ‘a’ and ‘b’ are consecutive integers. Assume that (a + b) is NOT
odd. If (a + b) is not odd then no integer k such that (a + b) = 2k + 1, but a + b = a
(a + 1) = 2a + 1. Shown that (a + b) ≠ 2k + 1 but (a + b) = 2a + 1. By default (a +
b). Therefore, the assumption that (a + b) is NOT odd is FALSE.

25
SUBJECT: GEED 10053: MATHEMATICS IN THE MODERN WORLD
PREPARED BY: Assoc. Prof. CHERRY E. ANGELES & WELFREDO B. QUEREMIT, LPT
PROOF BY INDUCTION

It is a method to show an infinite number of facts by showing some


specific case holds, and then using the assumption that the proposition is true for
some value of ‘n’, that is the proposition is also true for ‘n + 1. That is:
1. Show that the proposition from P(x) is true for some basis case.
2. Assuming that P(n) is true for some n show that this implies that
P(n + 1) is true.
3. It follows that the propositional form P(x) is true for all n greater or
equal to basis case.
Proof:
Let the propositional form F(x) be true when the sum of ‘x’ and its
successor is odd. Therefore, consider F(1); the sum 1 + 2 = 3 is odd, can
shows there exists k element of Z : 2k + 1 = 3, namely when k = 1.
Assume that F(x) is true for some x. Then x + (x + 1) is odd.
Notice x + 1 + ( x + 1 + 1) = (x + 1) + (x + 2) = F(x + 1).
Claim: since x + (x + 1) is odd adding 2 to this value again gives an odd
number.
Summary:

Direct Proof Steps:

1. Assume the statement p is true.

2. Use what we know about p and other facts as necessary to deduce

that another statement q is true, that is show p ⇒ q is true.

Proof by Contradiction: Steps

1. Assume your statement to be false.

2. Proceed as you would with a direct proof.

3. Come across a contradiction.


4. State that because of the contradiction, it can't be the case that the
statement is false, so it must be true.

Proof by Induction: Steps:

1. State what we want to prove e: P(n) for all n ≥ c, c ≥ 0 by induction on


n. The actual words that are used here will depend on the form of the
claim. (See the examples below.)
2. Base case: Prove P(c). This is usually easy to prove. It can be done by
considering cases or explicitly computing values.
3. Inductive hypothesis: Let k ≥ c be an arbitrary integer. Assume P(k) is
true. (Some induction proofs require that we assume P(n) is true for all c
≤ n ≤ k. That proof technique is called Strong Induction.)
26
SUBJECT: GEED 10053: MATHEMATICS IN THE MODERN WORLD
PREPARED BY: Assoc. Prof. CHERRY E. ANGELES & WELFREDO B. QUEREMIT, LPT
4. Inductive step Prove P(k + 1), assuming that P(k) is true. This is often
the most involved part of the proof. Apart from proving the base case, it is
usually the only part that is not boilerplate.
5. Apply the Induction rule: If have shown that P(c) holds and that for all
integers k ≥ c, assuming P(k) implies P(k + 1), then P(n) holds for all n ≥ c.
In symbols, this can be written as (P(c)&P(k) → P(k + 1)∀k ≥ c) →
P(n)∀n ≥ c.

Certainty has a perfect knowledge that has total security from error or mental state of
being without doubt. Objectively defined, certainty is total continuity and validity of all
foundational inquiry to the highest degree of precision. Something is certain only if no
skepticism occur.

Source: https://www.scribd.com/document/388414571/Intuition-Proof-and-Certainty

Read:
https://www.uni-siegen.de/fb6/phima/lehre/phima13/quellentexte/seminar_-
_hersh/hersh-chapter4.pdf

Watch:
https://www.youtube.com/watch?v=sFqEf36HJ4U

Activities/Assessments:

Use the three techniques of proving to prove the following:

1. Prove that if ‘m’ and ‘n’ are odd integers then ‘mn’ is also an odd integer.
2. Prove that if ‘n’ is an odd integer then n2 is also an odd integer
3. Let a, b and c be integers, prove that if ‘a’ divides ‘b’ and ‘a’ divides ‘c’ then ‘a’ also
divides b + c

References:

http://www-math.ucdenver.edu/~wcherowi/courses/m3000/lecture2.pdf

Unit 3 - Polya’s 4-Steps in Problem Solving

One of the foremost recent mathematicians to make a study of problem solving, a


Hungarian native, was George Polya (1887-1985).

Polya’s Four-Step Problem Solving Strategy

1. Understand the problem. Consider the following in following the step:


- Can you restate the problem in your own words?
- Can you determine what is known about these types of problem?

27
SUBJECT: GEED 10053: MATHEMATICS IN THE MODERN WORLD
PREPARED BY: Assoc. Prof. CHERRY E. ANGELES & WELFREDO B. QUEREMIT, LPT
- Is there missing information?
- What is the goal?

2. Devise a plan.Here are some example procedures:


- Make a list of the known information.
- Draw a diagram
- Make a table or chart
- Work backwards
- Look for a pattern
- Perform an experiment
- Guess at a solution and then check the result.

3. Carry out the plan.


- Work carefully
- Keep an accurate and neat record of all your attempts
- Realize that some of your initial plans will not work and that you may have to
devise another plan or modify your existing plan.

4. Review the solution.Check your solution.


- Ensure the solution is consistent with the facts of the problem.
- Interpret the solution in the contest of the problem.
- Ask yourself whether there are genralizations of the solution that couls apply to
other problem.

EXAMPLES
Solve “Twice the difference of a number and 1 is 4 more than that number. Find

` the number.” by applying Polya’s 4-steps

SOLUTION:

Step 1: Understand the Problem

Make sure that you read the question carefully several times. Since we
are looking for a number, we will let x = a number

Step 2: Devise a Plan (Translate)

Twice the difference of a number and 1 is 4 more than that number

2 (n – 1) = 4 + n

Hence the translated equation is 2(n – 1) = n + 4


Step 3: Carry out the Plan (Solve)
2(n – 1) = n + 4 - Given
2n – 2 = n + 4 - Distributive Property Over Multiplication
2n – n = 4 + 2 - Collect like terms
28
SUBJECT: GEED 10053: MATHEMATICS IN THE MODERN WORLD
PREPARED BY: Assoc. Prof. CHERRY E. ANGELES & WELFREDO B. QUEREMIT, LPT
n =6 - Simplify

Step 4: Look Back (Check, Review and Interpret the Solution)


The obtained value of a number is 6

Twice the difference of a number and 1 is 4 more than that number

2 (n – 1) = 4 + n
2 (6 – 1) = 4 + 6
2 (5) = 10
10 = 10

Final Answer: 6
Activities/Assessments:

1. Apply Polya’s strategy: A baseball team won out of their last four games. In how many
different orders could they have two wins and two losses in four games?

References:

http://www-math.ucdenver.edu/~wcherowi/courses/m3000/lecture2.pdf

Unit 4 - Problem Solving Strategies

Algorithm An algorithm is a step-by-step procedure that will alwyas produce a correct


solution. While an algorithm guarantees an accurate answer, it is not always the
best approach to problem solving.

Heuristics Heuristics are general strategies used to make quick short cut solutions to
problems that sometimes lead to solutions but sometimes lead to errors.
(Study.com)

Trial and Error A trial and error approach to problem solving involves trying a number of
different solutions and ruling out those that do not work.

Some other Problem Solving Strategies:


1. Draw a picture
2. Make a chart or list
3. Guess and check
4. Use a formula
5. Look for a pattern
6. Work backwards
7. Logical reasoning

EXAMPLES
29
SUBJECT: GEED 10053: MATHEMATICS IN THE MODERN WORLD
PREPARED BY: Assoc. Prof. CHERRY E. ANGELES & WELFREDO B. QUEREMIT, LPT
Use other problem-solving strategies to solve the problem below:

The Farmyard Problem: In the farmyard there are some pigs and some chickens. In
fact, there are 87 animals and 266 legs. How many pigs are there in the farmyard?

SOLUTION:

1. Guess and check. Let’s guess that there are 80 pigs. If there are, they will account
for 320 legs. Clearly, we’ve over-guessed the number of pigs. So maybe there are only 60 pigs.
Now 60 pigs would have 240 legs. That would leave us with 16 legs to be found from the
chickens. It takes 8 chickens to produce 16 legs. But 60 pigs plus 8 chickens is only 68 animals
so we have landed nearly 20 animals short.

Obviously, we haven’t solved the problem yet but we have now come to grips with some of the
important aspects of the problem. We know that there are 87 animals and so the number of pigs
plus the number of chickens must add up to 87. We also know that we have to use the fact that
pigs have four legs and chickens two, and that there have to be 266 legs altogether.

Guess and Improve. Supposed we guessed 60 pigs for a total of 240 legs. Now 60 pigs
imply 27 chickens, and that gives another 54 legs. Altogether then we’d have 294 legs at this
point.

Unfortunately, we know that there are only 266 legs. So, we’ve guessed too high. As pigs have
more legs than hens, we need to reduce the guess of 60 pigs. How about reducing the number
of pigs to 50? That means 37 chickens and so 200 + 74 = 274 legs.

We’re still too high. Now 40 pigs and 47 hens give 160 + 94 = 254 legs. We’ve now got too few
legs so we need to guess more pigs.

You should be able to see now how to oscillate backwards and forwards until you hit on the right
number of pigs. So, guess and improve is a method of solution that you can use on a number of
problems.

2. Making a Table illustrates this point. We’ll put a few values in and see what happens.

Pigs Chickens Pigs’ Legs Chicken’s Legs Total Difference

60 27 240 54 294 28

50 37 200 74 274 8

40 47 160 94 254 -12

41 46 164 92 256 -10

30
SUBJECT: GEED 10053: MATHEMATICS IN THE MODERN WORLD
PREPARED BY: Assoc. Prof. CHERRY E. ANGELES & WELFREDO B. QUEREMIT, LPT
From the table we can see that every time we change the number of pigs by one, we change
the number of legs by two. This means that in our last guess in the table, we are five pigs away
from the right answer. Then there have to be 46 pigs.

Geometry Problem

Some word problems can be solved by drawing or illustrating first the problem.
Visualizing the problem then put some labels will be a great help to understand and solve the
problem. Let’s consider the problem below.

A triangle has a perimeter of 50. If 2 of its sides are equal and the third side is 5 more
than the equal sides, what is the length of the third side?

SOLUTION:

Step 1: Assign variables:

Let x = length of the equal side


Sketch the figure

Step 2: Write out the formula for perimeter of triangle.

P = sum of the three sides

Step 3: Plug in the values from the question and from the sketch.

50 = x + x + x + 5

Combine like terms


50 = 3x + 5

Isolate variable x
3x = 50 – 5
3x = 45
x =15

Be careful! The question requires the length of the third side.


31
SUBJECT: GEED 10053: MATHEMATICS IN THE MODERN WORLD
PREPARED BY: Assoc. Prof. CHERRY E. ANGELES & WELFREDO B. QUEREMIT, LPT
The length of third side = 15 + 5 =20

Answer: The length of third side is 20

Activities/Assessments:

Solve the following by using other applicable strategies.


1. When 9 is subtracted from a number and then divided by 2, the answer is 4. What is
the number?
2. The length of a rectangle is twice that of the width. The perimeter of the rectangle is
24 cm. What is the width of the rectangle?
3. Marcus, Sally and Sammy decided to share 20 sweets. Marcus took 8 sweets and
Sally took three times as many as Sammy. How many sweets did Sammy receive?
Assignment

1. The length and width of a rectangle are 7 cm and (x - 8) cm respectively. Find the
value of x if the area of the rectangle is 42 cm2.
2. Diana buys 20 apples at x cents each and 40 oranges at x + 10 cents. She packs
them into bags containing 5 apples and 10 oranges and sells the bags for 20x cents
each. Find out the amount that Diana paid for each apple if she obtained a total of $24
from selling all the fruits.
References:

https://garyhall.org.uk/maths-problem-solving-strategies.html

Unit 5 - Mathematical Problems involving Patterns

Sequences A sequence is an ordered list of numbers. Each number in a sequence is called a


term of the sequence. The notation an, is used to denote the nth term of a
sequence. A formula that can be used to generate all the terms of a sequence is
called the nth-term formula.

Example: Consider the formula an= 2n2 + n, If we replace n by numbers 1,2,3 4, 5,


this generates the sequence 3, 10, 21, 36, 55. To find the 30th term, replace n with
30. So,
A30 = 2(30)2 + 30 = 1830

Difference Tables a difference table shows the difference between successive terms of a
sequence and in some cases it can be used to predict the next term in the
sequence.

32
SUBJECT: GEED 10053: MATHEMATICS IN THE MODERN WORLD
PREPARED BY: Assoc. Prof. CHERRY E. ANGELES & WELFREDO B. QUEREMIT, LPT
Example: The difference table for the sequence 5, 14, 27, 44, 65

Sequence 5 14 27 44 65

First difference 9 13 17 21

Second difference 4 4 4

In this table, the first difference are not all the same. Therefore, it is always
helpful to obtain the successive differences of the previous one.

Fibonacci Sequence Let Fn represent the nth Fibonacci number. Then the terms in the
Fibonacci sequence are given by the formula:

Fn = Fn-1 + Fn-2 where F1 = 1 and F2 = 1 for n ≥ 3.

Arithmetic Sequence and Series Word Problems


EXAMPLES
1. An amphitheater has 5 rows of seats with 30 seats in the first row., 32 in the
second. 34 in the third, and so on.
a. How many seats are there in the 50th row?
b. Find the seating capacity of the amphitheater
Solution:
The number of seats in the rows form an arithmetic sequence with n = 50, d = 2
and a1 = 30. To find the number of seats in the 50th row then,
a. an = a1 + (n-1)d
a50 = 30 + (50-1)2
a50 = 128 (seats in the last row, 50th row)
To find the seating capacity of the amphitheater, solve for S50, the sum seats
from the 1st row to the 50th row seat. Since the last term is known, a50 = 128 use the
formula.
b. Sn = n(a1+an)/2
S50 = 50(30+128)/2
S50 = 3 950 (Thus the amphitheater has 3,950 seats.)
Geometric Sequence and Series Word Problems
EXAMPLES
Mylo wishes to buy a cellphone worth P8,000 six weeks from now. He
starts saving P100 the first week, P200 the second week, P400 the third week
and so on.

33
SUBJECT: GEED 10053: MATHEMATICS IN THE MODERN WORLD
PREPARED BY: Assoc. Prof. CHERRY E. ANGELES & WELFREDO B. QUEREMIT, LPT
a. How much he saved at the 6th week?
b. What is the total amount he saved from the 1st week to the 6th week?
c. Will he be able to buy the cellphone after 6 weeks?
Solution:
a. Given: a1 = 100, r = 2, and n = 6
an = a1rn-1
a6 = 100(2)6-1
a6 = 3 200 (He saved P3,200 in 6th week)
b. Method 1 (Simple Addition)
S6 = P100 + P200 + P400 + P800 + P1,600 + P3,200 = P6 300
Thus, the total amount he saved from 1st week to 6th week is P6,300
Method 2 (Geometric Series Formula)
Sn = a1(1 – rn)/1 - r
S6 = 100(1 – 26)/1 – 2
S6 = -6,300/(-1)
S6 = 6 300
c. His money is not enough to buy a cellphone worth P8,000. Mylo need
additional P1,700 to avail that cellphone.
Activities/Assessments:

1. Use difference table to predict the next term in the sequence : 2, 7, 24, 59, 118, 207,…

2. Solve the following problems:


a. How many poles will be in a pile of telephone poles if there are 30 in the
bottom layer, 29 in the second, and so on, until there is one top layer?
b. Telephone poles are being stored in a pile with 25 poles in the first layer, 24 in
the second, and so on. If there are 12 layers, how many telephone poles does the pile
contain?
c. Beginning with 1, how may consecutive odd integers should be added to
obtain the sum of 441?

Assignment

Solve the following problems

34
SUBJECT: GEED 10053: MATHEMATICS IN THE MODERN WORLD
PREPARED BY: Assoc. Prof. CHERRY E. ANGELES & WELFREDO B. QUEREMIT, LPT
1. A virus reproduces by dividing into two, and after a certain growth period, it
divides into two again. As the virus continues to reproduce, it will continue to divide in
two. How many viruses will be in a system starting with a single virus AFTER 10
divisions?
a. Is the sequence arithmetic or geometric? Explain your answer.
b. Write out the sequence using blanks where appropriate. Fill in the first three terms.
c. Write an explicit formula for the sequence.
d. How many viruses will be in a system starting with a single virus AFTER 10 divisions?
e. Write your final answer as a sentence.

References:
Mathematics in the Modern World by Aufmann, Richard, et. al.
http://home.windstream.net/okrebs/page131.html#:~:text=A%20sequence%20is%20a%
20set,between%20two%20consecutive%20terms%20constant.

35
SUBJECT: GEED 10053: MATHEMATICS IN THE MODERN WORLD
PREPARED BY: Assoc. Prof. CHERRY E. ANGELES & WELFREDO B. QUEREMIT, LPT
PART 2: MATHEMATICS A S TOOL

LESSON 4: Data Management


Overview

This lesson presents the various types of statistical data and the different methods of
collecting, presenting and interpreting them. This will also provide a summary of characteristics
of a given set of data.

Learning Objectives:

After completion of the lesson, you should be able to:


1. Use a variety of statistical tools to process and manage data;
2. Use methods of linear regression and correlations to predict the value of a variable
given certain conditions;
3. Advocate the use of statistical data in decision making.

Course Materials

Unit 1 - Data Gathering and Organizing Data

Types of Data
1. Qualitative data are categorical data which takes the form of categories or attributes
such as sex, course, year level, race, religion, etc.
2. Quantitative data are obtained from measurements like heights, weights, ages, scores,
etc.

Measurement Scales
1. Nominal measurements are used for identification or classification purposes
2. Ordinal measurements do not only classify items. They also give the order or rank of
classes or objects.
3. Interval measurements make use of numbers assigned to items or objects.
4. Ratio measurements assigned as ratio of numbers being measured.

Sampling Techniques
1. Simple random sampling can be done through lottery. A random sample refers to a
limited number of individuals chosen from the population. Every individual has an equal
chance of being selected in the sample before the selection is done.
2. Stratified Random Sampling is done through dividing the population into categories or
strata and getting the members at random proportionate to each stratum or sub-group.
3. Systematic random sampling refers to the process of selecting every nth element in
the population until the desired sample size is acquired.
4. Cluster Sampling is an advantageous procedure when the population is spread out
over a wide geographical data.

Methods of Collecting Data


1. Direct or Interview Method
2. Indirect or questionnaire Method
36
SUBJECT: GEED 10053: MATHEMATICS IN THE MODERN WORLD
PREPARED BY: Assoc. Prof. CHERRY E. ANGELES & WELFREDO B. QUEREMIT, LPT
3. Registration Method
4. Other method: Observation, telephone

Organization of Data

After collecting the data, it is important to organize it for you to know the results or the
purpose. The second statistical process is the organization of data. The collected data can be
organize by using tables and frequency distribution table. A frequency distribution table shows a
clear and definite information about set of data. You can easily identified the scores with highest
or lowest frequencies.

Class Frequency

Class frequency refers to the number of observation belonging to a class interval.

The frequency distribution table below shows the scores of 60 students in 30-point quiz
in Math 7.

Class Interval (Scores) Number of Students (f)

28 – 29 1

26 – 27 3
24 – 25 3

22 – 23 3
20 – 21 6

18 – 19 6
16 – 17 8

14 – 15 6

12 – 13 10
10 - 11 14

N = 60

Methods of Presenting Data


1. Textular
2. Tabular
3. Semi-tabular
4. Graphical Presentation

37
SUBJECT: GEED 10053: MATHEMATICS IN THE MODERN WORLD
PREPARED BY: Assoc. Prof. CHERRY E. ANGELES & WELFREDO B. QUEREMIT, LPT
1. Line Graph – is used to represent changes (trend) in data over a period of
time.

The line graph below shows the sales (y) of a certain company in months (x)

2. Bar Graph – is used to compare two or more quantities.

The bar graph below shows the number of Mobile Subscribers in Different
Countries from 2006 to 2009.

3. Pie or Circle Graph – is used to show how all parts of something are related
to the whole.
38
SUBJECT: GEED 10053: MATHEMATICS IN THE MODERN WORLD
PREPARED BY: Assoc. Prof. CHERRY E. ANGELES & WELFREDO B. QUEREMIT, LPT
The circle graph below shows the favorite school lunch of some selected
students.

4. Pictograph – is used to represent a large quantity by using pictures or icons.

The pictograph below shows the happiness level of 5,600 employees in a certain
company.

39
SUBJECT: GEED 10053: MATHEMATICS IN THE MODERN WORLD
PREPARED BY: Assoc. Prof. CHERRY E. ANGELES & WELFREDO B. QUEREMIT, LPT
Interpretation of Data

After presenting the data, the next statistical process is the interpretation
of data. Interpreting data means translating the presented data through observation and
describing it. These will be the basis of the decision making and take considerations that
will help to improve something. The result of the interpretation will also give some
recommendations that will be beneficial to all people concerns in particular study.

Read:
https://www.slideshare.net/moneerah1/frequency-distribution-table

Activities/Assessments:

A. Classify the following as Quantitative or Qualitative. If a variable is quantitative,


Discrete variables further classify it as Discrete or Continuous. Continuous Variables
are countable in a would (literally) take
finite amount of time. 1. ages of congressman 6. gender forever to count. In fact,
For example, you you would get to “forever”
can count the 2. weight of newborn babies 7. marital status and never finish counting
change in your them. For example, take
pocket. You can 3. number of students in the auditorium 8. grade point average age. You can’t count “
count the money in age”. Why not? Because
your bank account. 4. brand of air conditioner 9. occupation it would literally take
You could also count forever. For example,
the amount of money 5. faculty rank 10. Nationality you could be:
in everyone’s bank 25 years, 10 months, 2
accounts. It might days, 5 hours, 4
take you a long time seconds, 4 milliseconds,
to count that last 8 nanoseconds, 99
item, but the point B. Match column A with column B. Write the letter of the correct answer. picosends…and so on.
is—it’s still countable. A B.
1. Data collected using face-to-face interviews or a. data Continuous data is data
Discrete variables that can take any value.
written questionnaires
can only be specific Height, weight,
values (typically 2. A small part of a group chosen to represent the b. survey
temperature and length
integers). For whole group. are all examples of
example: number of3. The information collected. c. experimentationcontinuous data.
4. The person who gathers data using observation
siblings. year of birth. d. sample Continuous variables.
method. These can have any
5. The method used by the physicist in collecting e. investigation value between a
Data. theoretical minimum and
maximum, like birth
C. Suggest a way to collect each set of data. weight, BMI,
temperature, neutrophil
1. Population of Region I, Region 2, and Region 3. count.
2. Effectiveness of the New Secondary
Curriculum
3. Heights of the 20 applicants
4. Number of customers in a restaurant from
40
SUBJECT: GEED 10053: MATHEMATICS IN THE MODERN WORLD
PREPARED BY: Assoc. Prof. CHERRY E. ANGELES & WELFREDO B. QUEREMIT, LPT
10:00 A.M. to 11:00 P.M.
5. Opinions of commuters about the color coding
Scheme

D. DIRECTION: Group the students below by preparing a table for ages and birth
month of the given data.

AGE (as of 1st


BIRTHDAY
MALE Friday of June)

Y mm/dd/yyyy

1 Abogado, Vincent Poul C. 11 10/18/2007

2 Aguilar, Steven John Beringuela 13 04/24/2006

3 Alejandro, Miguel Jonjie Gutang 11 07/31/2007

4 Alvior, Roisse Emmanuel Baldomar 11 08/22/2007

5 Aquino, Aaron Dave Villanueva 14 05/10/2005

6 Atayde, Mike Jermel Monares 12 10/13/2006

7 Bajao, Arman Sumaoy 11 09/25/2007

8 Basto, Geoff Andrei Valenzuela 12 04/30/2007

9 Cabañero, Jefferson Gianan 12 11/15/2006

10 Camarines, Nathaniel Malinao 13 02/23/2006

11 Consigna, Kenon Kurt Madolora 12 01/23/2007

12 dela Cruz, Alexander - 13 01/18/2006

13 De Paz, Reggie Papauran 11 09/09/2007

14 Ecalde Jr., Gilbert Delampasig 13 04/26/2006

15 Espiritu, Ritch Sainel D. 12 12/05/2006

16 Esteban, John Tristan Juliano 13 11/12/2005

17 Flores Jr., Renante Gersalia 12 01/24/2007

18 Icat, Clyde Vincent Fernandez 12 12/11/2006

19 Lejano, James Angelo Ventura 12 04/20/2007

41
SUBJECT: GEED 10053: MATHEMATICS IN THE MODERN WORLD
PREPARED BY: Assoc. Prof. CHERRY E. ANGELES & WELFREDO B. QUEREMIT, LPT
20 Magayones, Lorrens 15 12/14/2003

Assignment

E. 1. Construct a line graph given the set of data below

Year Island Middle School Eden Middle School


2008 700 500

2009 600 400


2010 700 600

2011 800 500


2012 900 600

2013 800 700

2. Construct a bar graph given the set of data below

Kind of animal Color of the bar Number of each animal

Snail red 8
Lizard blue 14

Rat green 12
Chameleon yellow 4

Frog pink 18

3. Construct a pictograph given the set of data below

Food Served Pizza Chicken Nuggets Grilled Cheese Hot Dogs

No. of Customers 40 35 20 25

2. Construct a circle/ pie graph given the set of data below

Activity Hours Degrees


School 6

Meal 2
Sleep 8

42
SUBJECT: GEED 10053: MATHEMATICS IN THE MODERN WORLD
PREPARED BY: Assoc. Prof. CHERRY E. ANGELES & WELFREDO B. QUEREMIT, LPT
Sports 3
Homework 2

Leisure Time 3
TOTAL 24 3600

References:

https://www.investopedia.com/terms/s/statistics.asp#:~:text=Statistics%20is%20a%20for
m%20of,Mean

Unit 2 - Measures of Central Tendency

Mean
The mean of n numbers is the sum of the numbers divided by n.

Mean 
x
n for ungrouped data

x
 fx
n for grouped data

Example.
Six students in Math class of 20 students received test grades of 92, 84, 65, 76, 88, and

90. Find the mean of test scores.Use x to represent mean.

Solution:

x
 x  92  84  65  76  88  90  82.5
n 6
Median

The median of a ranked list of n numbers is:


- the middle number of n if n is odd.
- the mean of the two middle numbers if n is even.

For grouped data:

43
SUBJECT: GEED 10053: MATHEMATICS IN THE MODERN WORLD
PREPARED BY: Assoc. Prof. CHERRY E. ANGELES & WELFREDO B. QUEREMIT, LPT
n 
  cf 
Md  u   2 i
 fi 
 
 
where u  the exact lower limit of the class interval containing the median
cf  cummulative frequency immediately below u
f i  frequency of the class interval containing the median

Example:

Find the mean of the data below:


1. 4,8,1,14,9,21,12
2. 46,23,92,89,77,108

Solution:

1. The list contains 7 numbers (odd). Ranking the numbers from smallest to largest gives,
1, 4, 8, 9, 12, 14, 21. The middle number is 9. Thus the median is 9.
2. The list contains 6 numbers(even). Ranking the numbers from smallest to largest
gives, 23, 46, 77, 89, 92, 108. The middle numbers are 77 and 89. Their mean is 83.
Thus the median is 83.

Mode

The mode of a list of numbers is the number that occurs most frequently.

Example:

Find the mode of the ff:


1. 18, 15, 25, 34, 15, 28, 36
2. 5, 6, 10, 20, 28, 31, 39, 47,55
3. 14, 19, 26, 19, 38, 52, 38, 70

Solution:

1. The number 15 occurs twice than other numbers, thus 15 is the mode.
2. There is no number appearing more than once in the list. Thus there is no mode.
3. The numbers 19 and 38 appear more frequent than other numbers. Thus 19 and 38
are the modes. We call it bi-modal.

Read:
https://statistics.laerd.com/statistical-guides/measures-central-
tendency-mean-mode-median-faqs.php

44
SUBJECT: GEED 10053: MATHEMATICS IN THE MODERN WORLD
PREPARED BY: Assoc. Prof. CHERRY E. ANGELES & WELFREDO B. QUEREMIT, LPT
EXAMPLES for Grouped Data

Compute for the mean, median and mode of given set of grouped data:

CLASS CLASS FREQUENCY fxm <cf


INTERVALS MARK (Xm) (f)

46 – 50 48 3 144 20
41 – 45 43 3 129 17

36 – 40 38 3 114 14

31 – 35 33 5 165 11
26 – 30 28 2 56 6

21 – 25 23 4 92 4
i =5 n= 20 ∑fxm = 608

Solution:
a. To find the mean of grouped data we will use the formula:
Mean (xe) = ∑fxm/n
where:
(xe) – mean
∑fxm – is the sum of the product of the frequency (f) and the class mark or
the class midpoint (xm)
n – is the total number of cases or frequencies.
Given:
∑fxm = 608
n = 20

Mean (xe) = ∑fxm/n = 608/20 = 30.4

b. To find the median of grouped data we will use the formula:


Median (x) = (n/2) – cfb)/fm]i + xLB
where:
(x) - median
cfb – is the cumulative frequency before the median class
fm – frequency of the median class
i – size of the class interval or the class width.
n – is the total number of cases or frequencies.
xLB – lower boundary of the median class
Given:
n/2 = 20/2 = 10th score the largest <cf after 10 is 11,
thus the Median class is 31 – 35
cfb = 6
fm = 5
i=5
n =20

45
SUBJECT: GEED 10053: MATHEMATICS IN THE MODERN WORLD
PREPARED BY: Assoc. Prof. CHERRY E. ANGELES & WELFREDO B. QUEREMIT, LPT
xLB = 30.5
Median (x) = (n/2) – cfb)/fm]i + xLB = (20/2) – 6)/5]5 + 30.5 = 34.5
c. To find the mode of grouped data we will use the formula:
Mode ( )= d1/(d1 + d2)]i + xLB
where:
( ) - mode
i – size of the class interval or the class width.
n – is the total number of cases or frequencies.
d1 – difference between the highest frequency and the frequency below it
d2 – difference between the highest frequency and the frequency above it
xLB – lower boundary of the modal class
Given:
Modal class: 31 – 35
d1= 5 – 2 = 2
d2= 5 – 3 = 2
i=5
n =20
xLB = 30.5
Mode ( )= d1/(d1 + d2)]i + xLB= 3/(3 + 2)]5 + 30.5 = 33.5

Activities/Assessments:

Compute for the mean, median and mode of the following data

1.) 24, 31, 12, 38, 12, 15 4.) 85, 58, 72, 85, 46, 93
2.) 5, 28, 16, 32, 5, 16, 48, 29, 5, 35 5.) 92, 63, 22, 80, 63, 71, 44, 35
3.) 53, 13, 34, 41, 26, 61, 34, 13, 69 6.) 39, 82, 74, 96, 64, 52, 74

Assignment

Compute for the mean, median and mode of the following data

CLASS FREQUENCY CLASS fxm <cf


INTERVALS (f) MARK (Xm)

45 – 49

40 – 44
35 – 39

30 – 34
25 – 29

20 – 24
15 – 19

i = n= ∑fxm =

46
SUBJECT: GEED 10053: MATHEMATICS IN THE MODERN WORLD
PREPARED BY: Assoc. Prof. CHERRY E. ANGELES & WELFREDO B. QUEREMIT, LPT
References:

https://www.investopedia.com/terms/s/statistics.asp#:~:text=Statistics%20is%20a%20for
m%20of,Mean

Mathematics in the Modern World by Aufmann, Richard, et. al.

General Statistics Made Simple by Nocon, Ferdinand, et. al.

Unit 3 - Measures of Dispersion

The Range
The range of a set of data values is the difference between the greatest data value and
the least data value.

The Average Deviation

Average Deviation (AD) – is the average of the sum of absolute values of the
deviation of scores from the mean score.

The Standard Deviation


1. Standard Deviation of population (σ)

  (x  ) 2

n where μ is the population mean


n is the population size
2. Standard Deviation of sample (s)

s
 ( x  x) 2

n 1

The Variance
The variance id the square of standard deviation. The standard deviation and variance
are the most reliable measures of variability(dispersion).

Example:

Score Deviation from Mean Squared Standard Deviation from


X Mean
( x  x)
( x  x) 2
7 7 - 10 = -3 (-3)2 = 9
8 8 -10 = -2 (-2)2 = 4
10 10 - 10 = 0 (0)2 = 0
12 12 - 10 = 2 (2)2 = 4
13 13 - 10 = 3 (3)2 = 9
 x  50  ( x  x) 2 = 26

47
SUBJECT: GEED 10053: MATHEMATICS IN THE MODERN WORLD
PREPARED BY: Assoc. Prof. CHERRY E. ANGELES & WELFREDO B. QUEREMIT, LPT
50
x  10
5

s
 ( x  x) 2

n 1
26
s
4
s  2.55 and the variance s2 = 6.50.

Ungrouped Data

EXAMPLES

1. The ages of children who received the DENGVAXIA are as follows: 9, 12, 7, 5,
2, 3. Find the range, average deviation, variance, and standard deviation

Solutions:
Arrange the scores in ascending order: 2, 3, 5, 7, 9, 12

a. Range (R) = H.S – L.S = 12 – 2 = 10

b. Average Deviation (AD) = (∑/x - xe/)/n

Step 1. Find the mean (xe)


Mean (xe) = ∑x/n = 38/6 = 6. 33 or 6

Step 2. Add the row /x - xe/. Find the difference of each score from the
mean and get their absolute values. Add all the values in row
/x - xe/. That is ∑/x - xe/
Scores (x) 2 3 5 7 9 12 ∑x = 38
/x - x⧠/ 4 3 1 1 3 6 ∑/x - x⧠/ = 18

Step 4. Substitute to the formula


Average Deviation (AD) = (∑/x - xe/)/n = 18/6 = 3

c. Variance (σ2) and Standard Deviation (σ)

Variance (σ2) = (∑(x - xe)2]/n and Standard Deviation (σ) = ∑(x - xe)2]/n
Step 1. Find the mean (xe)
Mean (xe) = ∑x/n = 38/6 = 6. 33 or 6

48
SUBJECT: GEED 10053: MATHEMATICS IN THE MODERN WORLD
PREPARED BY: Assoc. Prof. CHERRY E. ANGELES & WELFREDO B. QUEREMIT, LPT
Step 2. Add the row (x - xe)2. Square the difference between of each score
from the mean. Add all the values in row (x - xe)2, that is the
∑(x - xe)2.

Scores (x) 2 3 5 7 9 12 ∑x = 38
/x - x⧠/ 16 9 1 1 9 36 ∑(x - x⧠)2 = 72

Step 4. Substitute to the formula: Variance (σ2) = (∑(x - xe)2]/n and get the
square root of the variance to get the Standard Deviation (σ)

(σ2) = ∑(x - xe)2]/n = 72/6 = 12 and (σ) = 12 = 3.46 or 4

2. The ages of five parents elected as PTA Officers are: 48, 63, 56, 48 and 79.
Find the range, average deviation, variance, and standard deviation
.
Solutions:

Arrange the scores in ascending order: 48, 48, 56, 63, 79

a. Range (R) = H.S – L.S = 79 – 48 = 31

b. Average Deviation (AD) = (∑/x - xe/)/n

Step 1. Find the mean (xe)


Mean (xe) = ∑x/n = 294/5 = 58.8 or 59

Step 2. Add the row /x - xe/. Find the difference of each score from the
mean and get their absolute values. Add all the values in row
/x - xe/. That is ∑/x - xe/
Scores (x) 48 48 56 63 79 ∑x = 294
/x - x⧠/ 11 11 3 4 20 ∑/x - x⧠/ = 49

Step 4. Substitute to the formula


Average Deviation (AD) = (∑/x - xe/)/n = 49/5 = 9.8 or 10

c. Variance (σ2) and Standard Deviation (σ)

Variance (σ2) = (∑(x - xe)2]/n and Standard Deviation (σ) = ∑(x - xe)2]/n

Step 1. Find the mean (xe)


49
SUBJECT: GEED 10053: MATHEMATICS IN THE MODERN WORLD
PREPARED BY: Assoc. Prof. CHERRY E. ANGELES & WELFREDO B. QUEREMIT, LPT
Mean (xe) = ∑x/n = 294/5 = 58.8 or 59

Step 2. Add the row (x - xe)2. Square the difference between of each score
from the mean. Add all the values in row (x - xe)2, that is the
∑(x - xe)2.

Scores (x) 48 48 56 63 79 ∑x = 38
/x - x⧠/ 121 121 9 16 400 ∑(x - x⧠)2 = 667

Step 4. Substitute to the formula: Variance (σ2) = (∑(x - xe)2]/n and get the
square root of the variance to get the Standard Deviation (σ)

(σ2) = ∑(x - xe)2]/n = 667/5 = 133.4 and (σ) = 133.4 = 11.55 or 12

Grouped Data

EXAMPLES
Compute for the range, average deviation, variance and standard
deviation of given set of grouped data:
CLASS FREQUENCY CLASSMARK fxm /Xm - xe/ f/Xm - xe/ (Xm - xe) 2 f(Xm - xe) 2
INTERVALS (f) (Xm)
46 – 50 3 48 144 18 54 324 972
41 – 45 3 43 129 13 39 169 507
36 – 40 3 38 114 8 24 64 192
31 – 35 5 33 165 3 15 9 45
26 – 30 2 28 56 2 4 4 8
21 – 25 4 23 92 7 28 49 196
i =5 n = 20 ∑fxm = 608 ∑f/Xm - x⧠/ = 164 ∑ f(Xm - x⧠) 2 = 1920

Solution:

a. Range (R) = Upper boundary – Lower Boundary = 50.5 – 20.5 = 30

b. Average Deviation

Step 1. Find the mean of grouped data we will use the formula:

Mean (xe) = ∑fxm/n = 608/20 = 30.4 or 30

Step 2. Add the column /Xm - xe/. Find the absolute values of difference of each
score in classmark column from the mean.

Step 3. Add the column f/Xm - xe/. Multiply each frequency to the result in step 2.

Step 4. Get ∑f/Xm - xe/ by adding all the values in column of f/Xm - xe/
50
SUBJECT: GEED 10053: MATHEMATICS IN THE MODERN WORLD
PREPARED BY: Assoc. Prof. CHERRY E. ANGELES & WELFREDO B. QUEREMIT, LPT
Step 5. Substitute to the formula Average Deviation
(AD) = (∑f/x - xe/)/n = 164/20 = 8.2

a. Variance and Standard Deviation

Step 1. Find the mean of grouped data we will use the formula:

Mean (xe) = ∑fxm/n = 608/20 = 30.4 or 30

Step 2. Add the column (Xm - xe) 2. Square the difference of each score in
class mark column from the mean.

Step 3. Add the column f(Xm - xe) 2. Multiply each frequency to the result in step 2.

Step 4. Get ∑f(Xm - xe) 2 by adding all the values in column of f/(Xm - xe) 2

Step 5. Substitute to the formula: Variance (σ2) = (∑f(x - xe)2]/n and get the
square root of the variance to obtain the Standard Deviation (σ)

(σ2) = ∑f(x - xe)2]/n = 1920/20 = 96 and (σ) = 96 = 9.79 or 10

Activities/Assessments:

Compute for the range, average deviation, variance and standard deviation.

1.) 24, 31, 12, 38, 12, 15 4.) 85, 58, 72, 85, 46, 93
2.) 5, 28, 16, 32, 5, 16, 48, 29, 5, 35 5.) 92, 63, 22, 80, 63, 71, 44, 35
3.) 53, 13, 34, 41, 26, 61, 34, 13, 69 6.) 39, 82, 74, 96, 64, 52, 74

Assignment

Compute for the range, average deviation, variance and standard deviation.

CLASS FREQUENCY CLASS fxm <cf


INTERVALS (f) MARK (Xm)
45 – 49 3
40 – 44 5
35 – 39 7
30 – 34 10
25 – 29 7
20 – 24 5
15 – 19 3
i = n= ∑fxm =

51
SUBJECT: GEED 10053: MATHEMATICS IN THE MODERN WORLD
PREPARED BY: Assoc. Prof. CHERRY E. ANGELES & WELFREDO B. QUEREMIT, LPT
References:

https://www.investopedia.com/terms/s/statistics.asp#:~:text=Statistics%20is%20a%20for
m%20of,Mean

Unit 4 - Measures of Relative Position

Percentiles(P10, P20, P30, …)


The value x is called the pth percentile of a data set provided p% of the data values are
less than x.

Quartiles(Q1 Q2 Q3 Q4)
The three numbers Q1, Q2, Q3 that partition a ranked data into four equal groups are
called quartiles of the data. The following are the procedures for finding quartiles:
1. Rank the data.
2. Find the median in the data. This is the second quartile.
3. The first quartile is the median of the data values less than Q2. The third quartile Q3 is
the median of the data values greater than Q2.

Deciles(D1, D2, D3…, D9)


It is a measure of location that divides the distribution into 10 equal parts. D1 is 1/0th
score found in the distribution, D2 is 1/5th score, D3 is the 3/10th score and so on.

Read:
http://www.tihe.org/courses/it133/IT%20133%20Lectures/IT133%20-
%20Lecture%2004.pdf

EXAMPLES
Compute for the Q1, Q3, D3, D7, P75 and P90 of given set of grouped data:

CLASS FREQUENCY <cf


INTERVALS (f)
46 – 50 3 20
41 – 45 3 17
36 – 40 3 14
31 – 35 5 11
26 – 30 2 6
21 – 25 4 4
i =5 n = 20

Solution:

In determining the measure of location, we will apply on how to compute the


median in the same manner.

52
SUBJECT: GEED 10053: MATHEMATICS IN THE MODERN WORLD
PREPARED BY: Assoc. Prof. CHERRY E. ANGELES & WELFREDO B. QUEREMIT, LPT
a. For Q1 we will use the formula Q1 = (n/4) – cfq1)/fq1]i + xLB

Given:
Q1 class = n/4 = 20/4 = 5th score, 26 – 30, cfq1 = 4, fq1 = 3, xLB = 25.5 and i = 5
substitute the given data to the formula we have:

Q1 = (n/4) – cfq1)/fq1]i + xLB = (20/4) – 4)/2]5 + 25.5 = 28

b. For Q3 we will use the formula Q3 = (3n/4) – cfq3)/fq3]i + xLB

Given:
Q3 class = 3n/4 = 3(20)/4 = 15th score, 41 – 45, cfq3 = 14, fq3 = 2, xLB = 40.5 and i = 5
substitute the given data to the formula we have:

Q3 = (3n/4) – cfq3)/fq3]i + xLB = (3)(20)/4) – 14)/2]5 + 40.5 = 43

c. For D3 we will use the formula D3 = (3n/10) – cfd3)/fd3]i + xLB

Given:
D3 class = 3n/10 = 3(20)/10 = 6th score, 26 – 30, cfd3 = 4, fd3 = 2, xLB = 25.5 and i = 5
substitute the given data to the formula we have:

D3 = (3n/10) – cfd3)/fd3]i + xLB = (3)(20)/10) – 4)/2]5 + 25.5 = 30.5

d. For D7 we will use the formula D7 = (7n/10) – cfd7)/fd7]i + xLB

Given:
D7 class = 3n/10 = 7(20)/10 = 14th score, 36 – 40, cfd7 = 11,fd7 = 3,xLB = 35.5 and i = 5
substitute the given data to the formula we have:

D7 = (7n/10) – cfd7)/fd7]i + xLB = (7)(20)/10) – 11)/3]5 + 35.5 = 40.5

e. For P75 we will use the formula P75 = (75n/100) – cfp75)/fp75]i + xLB

Given:
P75 class = 75n/100 = 75(20)/100 = 15th score, 41 – 45, cfp75 = 14, fp75 = 3,
xLB = 40.5 and i = 5
substitute the given data to the formula we have:

P75 = (75n/100) – cfp75)/fp75]i + xLB = (75)(20)/100) – 14)/3]5 + 40.5 = 42.17

f. For P90 we will use the formula P90 = (90n/100) – cfp90)/fp90]i + xLB

Given:
P90 class = 90n/100 = 90(20)/100 = 18th score, 41 – 45, cfp90 = 14, fp90 = 3,
xLB = 40.5 and i = 5

substitute the given data to the formula we have:

P90 = (90n/100) – cfp90)/fp90]i + xLB = (90)(20)/100) – 14)/3]5 + 40.5 = 47.17

53
SUBJECT: GEED 10053: MATHEMATICS IN THE MODERN WORLD
PREPARED BY: Assoc. Prof. CHERRY E. ANGELES & WELFREDO B. QUEREMIT, LPT
Activities/Assessments:

Compute for the Q1, Q3, D4, D6, P45 and P80 of given set of grouped data:
CLASS FREQUENCY <cf
INTERVALS (f)
45 – 49 3
40 – 44 5
35 – 39 7
30 – 34 10
25 – 29 7
20 – 24 5
15 – 19 3
i = n=

References:

https://www.investopedia.com/terms/s/statistics.asp#:~:text=Statistics%20is%20a%20for
m%20of,Mean

Unit 5 - Probabilities and Normal Distribution

Probability is used to quantify the likelihood, or chance, that an outcome of a random


experiment will occur.

A probability model is used to calculate this proportion under reasonable assumptions for the
manner in which the sample is selected. This is fortunate because we do not want to attempt to
sample from an infinite series of lots.

Probability (P)

54
SUBJECT: GEED 10053: MATHEMATICS IN THE MODERN WORLD
PREPARED BY: Assoc. Prof. CHERRY E. ANGELES & WELFREDO B. QUEREMIT, LPT
A random experiment can result in one of the outcomes {a, b, c, d} with
probabilities 0.1, 0.3, 0.5, and 0.1, respectively. Let A denote the event {a,
b}, B the event {b, c, d}, and C the event {d}.Then,
P(A) = 0.1 = 0.3 = 0.4
P(B) = 0.3 + 0.5 + 0.1 = 0.9
P(C) = 0.1
Also, P(A’) = 0.6 and P(B’) = 0.1 and P(C”) =0.9

If one wafer is selected randomly from this process and the location is inspected, what is
the probability that it contains no particles?

P(0) = 0.40

What is the probability that a wafer contains three or more particles in the inspected
location?

Let E denote the events of wafer contains three or moreparticles

P(E) = 0.10 + 0.05 + 0.10 = 0.25

55
SUBJECT: GEED 10053: MATHEMATICS IN THE MODERN WORLD
PREPARED BY: Assoc. Prof. CHERRY E. ANGELES & WELFREDO B. QUEREMIT, LPT
Suppose that a batch contains six parts with part numbers {a, b, c, d, e, f}.
*(It has 30 outcomes)
Suppose that two parts are selected without replacement.
Let E denote the event that the part number of the first part selected is a.
Then E can be written as E = {ab, ac, ad, ae, af}.
P(E) = 5/ 30 = 1/6
Also, if E2 denotes the event that the second part selected is a,
E2 = {ba, ca, da, ea, fa} and with equally likely outcomes
P(E2) = 5/30 or 1/6

NORMAL DISTRIBUTION

A normal distribution forms a bell-shaped curve that is symmetric about a vertical line
through the mean of the data.

Properties:
1. Th graph is symmetric about a vertical line through the mean of the distribution.
2. The mean, median and mode are equal.
3. The y-value of each point on the curve is the percent of the data at the corresponding
x-value.
4. Areas under the curve that are symmetric about the mean are equal.
5. The total area under the curve is 1.

The Standard Normal Distribution

The standard normal distribution is the normal distribution that has mean of 0 and a
standard deviation of 1. Using the z-score formula,

56
SUBJECT: GEED 10053: MATHEMATICS IN THE MODERN WORLD
PREPARED BY: Assoc. Prof. CHERRY E. ANGELES & WELFREDO B. QUEREMIT, LPT
x xx
zx  or z x 
 s

Source:http://www.stat.yale.edu/Courses/1997-98/101/normal.htm

Activities/Assessments:

1. A random experiment can result in one of the outcomes {a, b, c, d} with probabilities 0.1, 0.3,
0.5, and 0.1, respectively.

Let A denote the event {a, b}, B the event {b, c, d}, and C the event {d}.

a. P(A) = _________
b. P(B) = _________
c. P(C) = _________
d. P(A’) = ________
e. P( A ∩ B) = ________
f. P( A U C) = __________
g. P(A U B) = _________
h. P( A U C)’ = ________

2. Samples of furniture are analyzed for surface and edge finish. The results from 303 samples
are summarized as follows:

Edge Finish

Surface Finish Excellent Good

Excellent 10 2

Good 10 8

57
SUBJECT: GEED 10053: MATHEMATICS IN THE MODERN WORLD
PREPARED BY: Assoc. Prof. CHERRY E. ANGELES & WELFREDO B. QUEREMIT, LPT
Let A denote the event that a sample has excellent edge finish, and let B denote the
event that a sample has excellent surface finish.

Determine the probability of polycarbonate plastic are

1. Excellent surfcae finish = ___________


2. Neither excellent edge finish nor excellent surface fiish = _______
3. Good edge finish = __________
4. Good surface finish and excellent edge = _______

5. Either good surface finish or good edge finish = _______

3. A soda machine disperses soda into 12-ounce cups. Tests show that the actual amount of
soda dispersed is normally distributed, with a mean of 11.5 oz. And a standard deviation of 0.2
oz.
a. What percent of cups will receive less than 11.25 oz. of soda?
b. What percent of cups will receive between 11.2 oz. And 11.55 oz. Of soda?
c. If a cup is filled at random, what is the probability that the machine will overflow
the cup?

References:
Mathematics in the Modern World by Aufmann, Richard, et. al.
General Statistics Made Simple by Nocon, Ferdinand, et. al.
https://www.investopedia.com/terms/s/statistics.asp#:~:text=Statistics%20is%20a%20for
m%20of,Mean

UNIT 6: Linear Regression and Correlation Analysis

Regression Analysis
The collection of statistical tools that are used to model and explore relationships
between variables that are related in nondeterministic manner is called regression analysis.
Basics of regressions
Which is the RESPONSE and which is the PREDICTOR?
The response or dependent variable varies with different values of the regressor/predictor.
The predictor values are fixed: we observe the response for these fixed values
The focus is in explaining the response variable in association with one or more predictors

SIMPLE LINEAR REGRESSION


Our goal is to find the best line that describes a linear relationship
Basic idea:
a. Plot observations (X, Y)
b. Find best line that follows plotted points

58
SUBJECT: GEED 10053: MATHEMATICS IN THE MODERN WORLD
PREPARED BY: Assoc. Prof. CHERRY E. ANGELES & WELFREDO B. QUEREMIT, LPT
c. Determine simple linear regression model

The case of simple linear regression considers a single regressor or predictor x and a
dependent or response variable Y.
To estimate (β0, β1) we find values that minimize squared error using least square estimates.

59
SUBJECT: GEED 10053: MATHEMATICS IN THE MODERN WORLD
PREPARED BY: Assoc. Prof. CHERRY E. ANGELES & WELFREDO B. QUEREMIT, LPT
We will fit a simple linear regression model to the oxygen purity data
in Table 11-1. The following quantities may be computed:

*Note: Use of SPSS will follow for more discussion.


60
SUBJECT: GEED 10053: MATHEMATICS IN THE MODERN WORLD
PREPARED BY: Assoc. Prof. CHERRY E. ANGELES & WELFREDO B. QUEREMIT, LPT
Correlation means association - more precisely it is a measure of the extent to which two
variables are related. There are three possible results of a correlational study: a positive
correlation, a negative correlation, and no correlation.
A positive correlation is a relationship between two variables in which both variables move
in the same direction. Therefore, when one variable increases as the other variable increases,
or one variable decreases while the other decreases
A negative correlation is a relationship between two variables in which an increase in one
variable is associated with a decrease in the other.
A zero correlation exists when there is no relationship between two variables.
Scatter Plot
A correlation can be expressed visually. This is done by drawing a scatter plot. It is a graphical
display that shows the relationships or associations between two numerical variables (or co-
variables), which are represented as points (or dots) for each pair of score. It also indicates the
strength and direction of the correlation between the co-variables.

*Note: Use of SPSS will follow for the computation of r.

Activities/Assessments:

1. An article in the Journal of Environmental Engineering(Vol. 115, No. 3, 1989, pp. 608–619)
reported the results of a study on the occurrence of sodium and chloride in surface streams in
central Rhode Island. The following data are chloride concentration y (in milligrams per liter) and
roadway area in the watershed x (in percentage).

(A) Draw a scatter diagram of the data. Does a simple linear regression model seem
appropriate here? (b) Fit the simple linear regression model using the method of least squares.
(c) Estimate the mean chloride concentration for a watershed that has 1% roadway area.
(d) Find the fitted value corresponding to x = 0.47.

61
SUBJECT: GEED 10053: MATHEMATICS IN THE MODERN WORLD
PREPARED BY: Assoc. Prof. CHERRY E. ANGELES & WELFREDO B. QUEREMIT, LPT
2. As Earth’s population continues to grow, the solid waste generated by the population grows
with it. Governments must plan for disposal and recycling of ever growing amounts of solid
waste. Planners can use data from the past to predict future waste generation and plan for
enough facilities for disposing of and recycling the waste.

Given the following data on the waste generated in Florida from 1990-
1994, how can we construct a function to predict the waste that was generated in the
years 1995-1999? The scatter plot is shown in Figure 1.85.

Year Tons of Solid Waste


Generated (in thousands)

1990 19,358

1991 19,484
1992 20,293
1993 21,499
1994 23,561
a) Make a scatterplot of the data, letting x represent the number of years since 1990.
b) Use a graphing calculator to fit linear, quadratic, cubic, and power functions to the
data. By comparing the values of R 2 , determine the function that best fits the data.
c) Graph the function of best fit with the scatterplot of the data.
d) With each function found in part (b), predict the average tons of waste in 2000 and
2005, and determine which function gives the most realistic predictions.

3. Compute for the coefficient of correlation r. Plot the actual data and the model you selected
on the same graph. How closely does the model represent the data?

a. The numbers of insured commercial banks y (in thousands) in the United States for
the years 1987 to 1996 are shown in the table. (Source: Federal Deposit Insurance
Corporation).

Year 1987 1988 1989 1990 1991 1992 1993 1994 1995 1996
y 13.70 13.12 12.71 12.34 11.92 11.46 10.96 10.45 9.94 9.53

b. U.S. Farms. As the number of farms has decreased in the United States, the average
size of the remaining farms has grown larger, as shown in the table below.

Year Average Acreage Per


Farm
1910 139

62
SUBJECT: GEED 10053: MATHEMATICS IN THE MODERN WORLD
PREPARED BY: Assoc. Prof. CHERRY E. ANGELES & WELFREDO B. QUEREMIT, LPT
1920 149
1930 157
1940 175
1950 216
1959 303
1969 390
1978 449
1987 462
1997 487

c. Sports The winning times (in minutes) in the women’s 400-meter freestyle swimming
event in the Olympics from 1936 to 1996 are given by the following ordered pairs.
(1936,5.44) (1972, 4.32)
(1948,5.30) (1976, 4.16)
(1952,5.20) (1980, 4.15)
(1956, 4.91) (1984, 4.12)
(1960, 4.84) (1988, 4.06)
(1964, 4.72) (1992, 4.12)
(1968, 4.53) (1996, 4.12)

References:

http://www.math.uakron.edu/amc/DataAnalysis_Statistics/Data%20Analysis/Problems_Other/Dif
ferent%20Regression%20Problems.doc

https://www.datasciencecentral.com/profiles/blogs/difference-between-correlation-and-
regression-in-statistics

63
SUBJECT: GEED 10053: MATHEMATICS IN THE MODERN WORLD
PREPARED BY: Assoc. Prof. CHERRY E. ANGELES & WELFREDO B. QUEREMIT, LPT
LESSON 5: Linear Programming
Overview

Linear programming was developed during World War II, when a system with which to
maximize the efficiency of resources was of utmost importance. New war-related projects
demanded attention and spread resources thin. “Programming” was a military term that referred
to activities such as planning schedules efficiently or deploying men optimally. George Dantzig,
a member of the U.S. Air Force, developed the Simplex method of optimization in 1947 in order
to provide an efficient algorithm for solving programming problems that had linear structures.
Since then, experts from a variety of fields, especially mathematics and economics, have
developed the theory behind “linear programming” and explored its applications.

LEARNING OBJECTIVES:

After successful completion of this module, you should be able to:


1. Use mathematical concepts and tools in other areas. (S)
2. Support the use of mathematics in various aspects and endeavors. (V)

COURSE MATERIALS:

UNIT 1: Linear Inequalities

Linear Inequality in Two Variables

- whose standard forms are as follows:


Ax + By > C, Ax + By > C, Ax + By < C, and Ax + By < C,

The solution of a linear inequality in two variables like is a set of ordered pairs (x, y) that
produces a true statement when the values of x and y are substituted into the inequality.

Example

Is (1, 2) a solution to the inequality


2x+3y>12x+3y>1
2⋅1+3⋅2>?12⋅1+3⋅2>?1
2+5>?12+5>?1
7>1

The graph of an inequality in two variables is the set of points that represents all
solutions to the inequality. A linear inequality divides the coordinate plane into two
halves by a boundary line where one half represents the solutions of the inequality. The
boundary line is dashed for > and < and solid for ≤ and ≥. The half-plane that is a
solution to the inequality is usually shaded.
64
SUBJECT: GEED 10053: MATHEMATICS IN THE MODERN WORLD
PREPARED BY: Assoc. Prof. CHERRY E. ANGELES & WELFREDO B. QUEREMIT, LPT
Example

Graph the inequality y ≥ −x+1

Solution:

Graph the inequality as linear equation in two variables, we need to rewrite the inequality
as y = - x + 1. Solving for the intercepts we obtained (1,0) and (0,1). The next step is to
determine what half of the plane will shade.
Set (0,0) or the origin as the testing point in our original inequality we have:
y ≥ −x+1
0 ≥ −0 +1
0 ≥ 1 False
This tells us that the origin (0,0) is not included to the half plane that we need to shade.

Graph of linear inequality y ≥ −x+1. Take note that we used solid line because the
inequality symbol is ≥. However, if the inequality symbol like < or >, dashed or broken line
must be utilized.

The shaded region is the solution set of the inequality.


This lesson presents the various types of statistical data and the different methods of
collecting, presenting and interpreting them. This will also provide a summary of characteristics
of a given set of data.

Activities/Assessments:

Graph the following linear inequality and identify the solution set.

1. 3x + 4y < 12 4. y + (2/3)x < 12


2. 2x > 3y + 6 5. 5x – 20 < 10y
3. 10 – 2x > 4y 6. 2y > -16

Assignment
65
SUBJECT: GEED 10053: MATHEMATICS IN THE MODERN WORLD
PREPARED BY: Assoc. Prof. CHERRY E. ANGELES & WELFREDO B. QUEREMIT, LPT
Graph the following linear inequality and identify the solution set.

1. 4x - 3y < 12 4. y - (2/3)x < 12


2. 2x + 3y > 6 5. 5x + 20 < 10y
3. 20 + 2x > 4y 6. 2x > -16

References:
https://brilliant.org/wiki/linear-programming/

Unit 2: Geometry of Linear Programming Problems

Linear programming is the process of taking various linear inequalities relating to some
situation, and finding the "best" value obtainable under those conditions. A typical example
would be taking the limitations of materials and labor, and then determining the "best"
production levels for maximal profits under those conditions.

In "real life", linear programming is part of a very important area of mathematics called
"optimization techniques". This field of study (or at least the applied results of it) are used every
day in the organization and allocation of resources. These "real life" systems can have dozens
or hundreds of variables, or more. In algebra, though, you'll only work with the simple (and
graphable) two-variable linear case.

The general process for solving linear-programming exercises is to graph the


inequalities (called the "constraints") to form a walled-off area on the x,y-plane (called the
"feasibility region"). Then you figure out the coordinates of the corners of this feasibility region
(that is, you find the intersection points of the various pairs of lines), and test these corner points
in the formula (called the "optimization equation") for which you're trying to find the highest or
lowest value.

 Find the maximal and minimal value of z = 3x + 4y subject to the following


constraints:

The three inequalities in the curly braces are the constraints. The area of the plane that
they mark off will be the feasibility region. The formula "z = 3x + 4y" is the optimization equation.
I need to find the (x, y) corner points of the feasibility region that return the largest and smallest
values of z.

My first step is to solve each inequality for the more-easily graphed equivalent forms:

66
SUBJECT: GEED 10053: MATHEMATICS IN THE MODERN WORLD
PREPARED BY: Assoc. Prof. CHERRY E. ANGELES & WELFREDO B. QUEREMIT, LPT
It's easy to graph the system: Copyright © Elizabeth Stape006-2011 All Rights Reserved

To find the corner points -- which aren't always clear from the graph -- I'll pair the
lines (thus forming a system of linear equations) and solve:

y = –( 1/2 )x + 7 y = –( 1/2 )x + 7 y = 3x
y = 3x y=x–2 y=x–2
–( 1/2 )x + 7 = 3x –( 1/2 )x + 7 = x – 2
3x = x – 2
–x + 14 = 6x –x + 14 = 2x – 4
2x = –2
14 = 7x 18 = 3x
x = –1
2=x 6=x
y = 3(–1) = –3
y = 3(2) = 6 y = (6) – 2 = 4
corner point at (2, 6) corner point at (6, 4) corner pt. at (–1, –3)

So the corner points are (2, 6), (6, 4), and (–1, –3).

Somebody really smart proved that, for linear systems like this, the maximum and
minimum values of the optimization equation will always be on the corners of the feasibility
region. So, to find the solution to this exercise, I only need to plug these three points into
"z = 3x + 4y".
67
SUBJECT: GEED 10053: MATHEMATICS IN THE MODERN WORLD
PREPARED BY: Assoc. Prof. CHERRY E. ANGELES & WELFREDO B. QUEREMIT, LPT
(2, 6): z = 3(2) + 4(6) = 6 + 24 = 30
(6, 4): z = 3(6) + 4(4) = 18 + 16 = 34
(–1, –3): z = 3(–1) + 4(–3) = –3 – 12 = –15

Then the maximum of z = 34 occurs at (6 4),


and the minimum of z = –15 occurs at (–1 –3).

Activities/Assessments:

1. .Solve the following Linear Programming Problems

A farmer has recently acquired a 110 hectares piece of land. He has decided to grow
Wheat and barley on that land. Due to the quality of the sun and the region’s excellent climate,
the entire production of Wheat and Barley can be sold. He wants to know how to plant each
variety in the 110 hectares, given the costs, net profits and labor requirements according to the
data shown below:

Variety Cost (Price/Hec) Net Profit (Price/Hec) Man-days/Hec


Wheat 100 50 10
Barley 200 120 30

The farmer has a budget of US$10,000 and availability of 1,200 man-days during the planning
horizon. Find the optimal solution and the optimal value.

Assignment

Solve the following Linear Programming Problems

Consider this scenario: your school is planning to make toques and mitts to sell at the
winter festival as a fundraiser. The school’s sewing classes divide into two groups – one group
can make toques,the other group knows how to make mitts. The sewing teachers are also
willing to help out. Considering the number of people available and time constraints due to
classes, only 150 toques and 120 pairs of mitts can be made each week. Enough material is
delivered to the school every Monday morning to make a total of 200 items per week. Because
the material is being donated by community members, each toque sold makes a profit of $2 and
each pair of mitts sold makes a profit of $5.

In order to make the most money from the fundraiser, how many of each item should be made
each week?

References:
https://brilliant.org/wiki/linear-programming/

68
SUBJECT: GEED 10053: MATHEMATICS IN THE MODERN WORLD
PREPARED BY: Assoc. Prof. CHERRY E. ANGELES & WELFREDO B. QUEREMIT, LPT
UNIT 3: SIMPLEX METHOD

69
SUBJECT: GEED 10053: MATHEMATICS IN THE MODERN WORLD
PREPARED BY: Assoc. Prof. CHERRY E. ANGELES & WELFREDO B. QUEREMIT, LPT
70
SUBJECT: GEED 10053: MATHEMATICS IN THE MODERN WORLD
PREPARED BY: Assoc. Prof. CHERRY E. ANGELES & WELFREDO B. QUEREMIT, LPT
71
SUBJECT: GEED 10053: MATHEMATICS IN THE MODERN WORLD
PREPARED BY: Assoc. Prof. CHERRY E. ANGELES & WELFREDO B. QUEREMIT, LPT
72
SUBJECT: GEED 10053: MATHEMATICS IN THE MODERN WORLD
PREPARED BY: Assoc. Prof. CHERRY E. ANGELES & WELFREDO B. QUEREMIT, LPT
73
SUBJECT: GEED 10053: MATHEMATICS IN THE MODERN WORLD
PREPARED BY: Assoc. Prof. CHERRY E. ANGELES & WELFREDO B. QUEREMIT, LPT
74
SUBJECT: GEED 10053: MATHEMATICS IN THE MODERN WORLD
PREPARED BY: Assoc. Prof. CHERRY E. ANGELES & WELFREDO B. QUEREMIT, LPT
75
SUBJECT: GEED 10053: MATHEMATICS IN THE MODERN WORLD
PREPARED BY: Assoc. Prof. CHERRY E. ANGELES & WELFREDO B. QUEREMIT, LPT
Activities/Assessments:

Solve the following Linear Programming Problems Using Simplex Method

A farmer has recently acquired a 110 hectares piece of land. He has decided to grow
Wheat and barley on that land. Due to the quality of the sun and the region’s excellent climate,
the entire production of Wheat and Barley can be sold. He wants to know how to plant each
variety in the 110 hectares, given the costs, net profits and labor requirements according to the
data shown below:
Variety Cost (Price/Hec) Net Profit (Price/Hec) Man-days/Hec
Wheat 100 50 10
Barley 200 120 30

76
SUBJECT: GEED 10053: MATHEMATICS IN THE MODERN WORLD
PREPARED BY: Assoc. Prof. CHERRY E. ANGELES & WELFREDO B. QUEREMIT, LPT
The farmer has a budget of US$10,000 and availability of 1,200 man-days during the planning
horizon. Find the optimal solution and the optimal value.

Assignment

Solve the following Linear Programming Problems Using Simplex Method

Consider this scenario: your school is planning to make toques and mitts to sell at the
winter festival as a fundraiser. The school’s sewing classes divide into two groups – one group
can make toques,the other group knows how to make mitts. The sewing teachers are also
willing to help out. Considering the number of people available and time constraints due to
classes, only 150 toques and 120 pairs of mitts can be made each week. Enough material is
delivered to the school every Monday morning to make a total of 200 items per week. Because
the material is being donated by community members, each toque sold makes a profit of $2 and
each pair of mitts sold makes a profit of $5.

In order to make the most money from the fundraiser, how many of each item should be made
each week?

References:

https://brilliant.org/wiki/linear-programming/

77
SUBJECT: GEED 10053: MATHEMATICS IN THE MODERN WORLD
PREPARED BY: Assoc. Prof. CHERRY E. ANGELES & WELFREDO B. QUEREMIT, LPT
LESSON 6: THE MATHEMATICS OF FINANCE

Overview

Everybody uses money. Sometimes you work for your money and other times your
money works for you. For example, unless you are attending college on a full scholarship, it is
very likely that you and your family have either saved money or borrowed money, or both, to
pay for your education. When we borrow money, we normally have to pay interest for that
privilege. When we save money, for a future purchase or retirement, we are lending money to a
financial institution and we expect to earn interest on our investment. We will develop the
mathematics in this lesson to understand better the principles of borrowing and saving. These
ideas will then be used to compare different financial opportunities and make informed decisions.

Learning Objectives:

After successful completion of this module, you should be able to:


1. Differentiate simple interest from compound interest
2. Compute simple interest and compound interest
3. Differentiate credit loan from credit card.
4. Differentiate stocks, bonds and mutual funds.

Course Materials:

UNIT 1: Simple and Compound Interest

Simple Interest
This section will discuss about simple interest. This type of interest computed
based on the principal and is paid at the end of a specified period of time. The following formula
is used to compute this type of interest:
I = Prt

where:

P – principal invested

r – rate of interest (converted to its decimal equivalent)

t – time (expressed in years)

Example:

Find the interest paid by Ms. Rose on $15,000 that she borrowed for three years
at 6% simple interest.

78
SUBJECT: GEED 10053: MATHEMATICS IN THE MODERN WORLD
PREPARED BY: Assoc. Prof. CHERRY E. ANGELES & WELFREDO B. QUEREMIT, LPT
Solution:

Given: Computation:

P = $15,000 I = $15,000(0.06)(3) = $2,700

r = 6% = 0.06

t = 3 years Therefore, the interest paid by Ms. Rose

Required: Interest (I) is $2,700

Formula: I = Prt

Compound Amount and Compound Interest

Compound Amount

This is an accumulated amount composed of the principal and the compound


interest. This is denoted by F and can be computed using the following formula:

F = P(1 + i)n

where:

P – original principal

i = (r/m) – periodic rate of interest

n = t(m) – total number of conversion periods for the whole term

Compound Interest

This is an interest resulting from the periodic addition of simple interest to the
principal amount. This is denoted by I and can be computed using the following formula:

I=F–P or I = P[(1 + i)n – 1]

Example:

Determine the amount and interest if $10,500 is invested at 10% compounded


quarterly for three years.
79
SUBJECT: GEED 10053: MATHEMATICS IN THE MODERN WORLD
PREPARED BY: Assoc. Prof. CHERRY E. ANGELES & WELFREDO B. QUEREMIT, LPT
Solution:

Given:

P = $10,000 m = 4 (since the amount invested is compounded quarterly)

r = 6% = 0.06 i = r/m = 0.06/4 = 0.025

t = 3 years n = t(m) = 3(4) = 12

Required: F and I

Formulas: F = P(1 + i)n and I = F – P

Computation:

F = 10,000(1 + 0.025)12 = 10,000(1.344888824) = $14,121.33

I = $14,121.33 – $10,000 = $3,621.33

Thus, the compound amount is $14,121.33 and the compound interest is $3,621.33

Activities/Assessments:

Solve for the simple and compound interest

1. Determine the simple interest to be paid by Ana if she borrowed $1,600 at


6.25% for 7 months.
2. Find the compound amount and interest on $175,250 for 15 years and 6
months at 6.75% compounded quarterly.

ASSIGNMENT

Solve for the simple and compound interest

1. At what rate of interest did Mercy invest the amount of $23,500 for it to earn a
simple interest of $3,525 for 3 years.
2. Find the compound amount and interest on $14,700 for 5 years and 6 months at
12% compounded semiannually.

80
SUBJECT: GEED 10053: MATHEMATICS IN THE MODERN WORLD
PREPARED BY: Assoc. Prof. CHERRY E. ANGELES & WELFREDO B. QUEREMIT, LPT
References:

Link(s):
https://www.pearson.com/content/dam/one-dot-com/one-dot-com/us/en/higher-
ed/en/products-services/course-products/lial-applied-mathematics-info/pdf/LGR-Finite-Ch5.pdf

Book(s):
Mathematics of Investment by Cervillon, Carmelita C., et. al.

UNIT 2: Credit Cards and Consumer Loans

A credit card and a consumer loan are two different ways of borrowing money and they
provide different benefits. Which of the two is best suited for you depends on your need and
purchasing pattern.

A credit card provides a lot more flexibility in that you can decided how much you want
to borrow and how much you want to pay back every month. As long as you stay within your
credit limit you have the freedom to decided how much you want to borrow and how much to
pay back each month. However, flexibility can tempt some to spend beyond their means. A
credit card is therefore more demanding since you need to be in charge of your own spending
and what you owe.

A consumer loan is a good alternative to a credit card if you want predictability with
your monthly expenses. A consumer loan provides a set plan for your monthly down payments
which gives many a sense of security. You can arrive back from a vacation paid with a
consumer loans and not expect any surprises. You will simply start paying back a pre decided
amount each month.

Summed up a consumer loan is the best choice for large purchases that you plan to pay
down over a longer period of time, while a credit card is best for smaller purchases that can be
paid back relatively quick.

About consumer loans:

 Good for someone who wants to borrow a one-time amount that you pay back following
a set down payment plan.
 A consumer loan provides structure and predictability in your finances and you know
exactly how much you need to pay back each month.
 Lower interest than credit card debt.
 A consumer loan enables you to refinance smaller and more expensive loans.

About credit cards:

 Good for someone who prefers flexibility as it gives you the opportunity to borrow as you
go as long as you stay within your credit limit provided.
 Offers a no interest period between 30 and 52 days
81
SUBJECT: GEED 10053: MATHEMATICS IN THE MODERN WORLD
PREPARED BY: Assoc. Prof. CHERRY E. ANGELES & WELFREDO B. QUEREMIT, LPT
 You decided how much you want to pay within a certain time frame
 The card may give you additional benefits such as bonuses and discounts in stores,
restaurants, online and other partner benefits
 Most often a credit card includes travel and cancellation insurance

Read:
https://corporatefinanceinstitute.com/resources/knowledge/credit/consumer-loan/

Activities/Assessments:

Give some advantages of using credit loans and credit cards.

ASSIGNMENT

Search for the types of consumer loan. Explain each type.

UNIT 3: Stocks Bond and Mutual Funds

Stocks

Stocks are an investment in a company and are very common in the financial world. If
you own a stock, you are a part owner of the business. Stocks are more volatile than bonds,
meaning there typically is more risk, but the returns are frequently greater.

The value of stocks fluctuates, meaning at any point, your original investment could be
more or less than the stock is worth. In theory, the goal of buying stocks is to sell them at a
higher price than you bought them, but doing so is easier said than done. There are many
different strategies as to when to buy or sell stocks. A fiduciary can help you develop an
investment strategy based on your risk tolerance.

Another way to make money from stocks is by receiving dividends, which are a share of
the company’s profits. Not all stocks provide dividends.

Stocks are purchased inside a brokerage account, which is an account that allows you to
deposit money that is used by your broker to acquire different investments and stocks. Stocks
are sold in whole numbers of shares from one to hundreds of thousands of shares.

Bonds

One of the biggest differences between stocks, bonds, and mutual funds is how each of
them allows you to invest your money. For example, bonds are actually a loan to a company or
the government.

Each bond has a maturity date, which is when a bond is redeemed at its par or face
value. At the time of maturity, you—the bondholder— will receive the original amount of money
you invested back plus interest. Though less volatile than stocks, bondholders can lose money if
82
SUBJECT: GEED 10053: MATHEMATICS IN THE MODERN WORLD
PREPARED BY: Assoc. Prof. CHERRY E. ANGELES & WELFREDO B. QUEREMIT, LPT
the value of their bonds decrease. This can happen due to higher interest rates or if the credit
quality—a criteria that informs investors of the risk of default—of the bond decreases prior to
maturity.

Unlike stocks, the commission to buy a bond is hidden inside the price of the bond.
Typically, a bond purchaser pays a 2% commission to buy and another 2% to sell a bond. One
way to minimize this cost is to buy a bond that you expect to hold to maturity, which is the date
the bond officially ends. There is no commission to get your money back when the bond
matures.

Mutual Funds

A mutual fund is made up of a pool of money collected from many different investors for
the purpose of investing in stocks, bonds, real estate, or money market accounts.
A mutual fund can be a passive investment, meaning that those who invest do not have an
active strategy for buying and selling the investments in the fund. Instead, they try to imitate an
index, which is the number that refers to the value of the investments. Passive investors follow
the stock market indexes to help model their mutual funds. Dow Jones Industrial Average, S&P
500 Index, and Russell 2000 Index are great examples of stock market indexes.

Another kind of mutual fund is referred to as an active fund. Active funds are controlled
by a manager who actively strategizes and makes all the buy, hold, and sell decisions. The
manager of a mutual fund will design and maintain the fund to match its investment goals as
stated in the fund’s prospectus, a legal document that outlines details about the investments.
Always read the prospectus before buying a fund.

Some funds are loaded, meaning there is a sales charge to the broker who sold you the
fund, and some funds are no-load. Loaded funds do not necessarily perform better than no-load
funds, so always take the time to review your options. Funds also have an annual expense ratio
that reduces the investor’s return. An annual expense ratio of 0.05% is extremely low and an
expense ratio of 1.5% is very high.

Read:

https://finance.zacks.com/difference-between-stocks-bonds-mutual-funds-2329.html

Activities/Assessments:

On your own understanding, differentiate stocks, bonds and mutual funds.

ASSIGNMENT

1. How to compute the Bond Valuation?


2. Define the following:
a. Coupon Payment c. Purchase Price
b. Redemption Value d. Face Value

References:
83
SUBJECT: GEED 10053: MATHEMATICS IN THE MODERN WORLD
PREPARED BY: Assoc. Prof. CHERRY E. ANGELES & WELFREDO B. QUEREMIT, LPT
https://www.lorenzfinancialservices.com/investments/difference-between-stocks-bonds-
mutual-funds/
Mathematics of Investment by Cervillon, Carmelita C., pages 193 – 197.

84
SUBJECT: GEED 10053: MATHEMATICS IN THE MODERN WORLD
PREPARED BY: Assoc. Prof. CHERRY E. ANGELES & WELFREDO B. QUEREMIT, LPT
LESSON 7: APPORTIONMENT AND VOTING

Overview

One of the most precious rights in our democracy is the right to vote. We have elections
to select the president of the Philippines, senators and representatives, members of the cabinet,
baseball players to be inducted into the Baseball Hall of Fame, and even “best” performers to
receive Oscar and Grammy awards. There are many ways of making the final decision in these
elections, some simple, some more complex.
In this chapter we will look at several voting methods, the “fairness” of these methods,
how votes are apportioned or divided among voters or states, and the fairness of these
apportionments.

Learning Objectives:

After successful completion of this module, you should be able to:


1. Define Apportionment
2. Define Voting
3. Compute for weighted voting system

Course Materials:

UNIT 1: Introduction to Apportionment

Apportionment involves dividing something up, just like fair division. In fair division we
are dividing objects among people while in apportionment we are dividing people among places.
Also like fair division, the apportionment processes that are widely used do not always give the
best answer, and apportionment is still an open field of mathematics.
Example of how apportionment can be used is to assign a group of new fire fighters to
the fire stations in town in an equitable way. Overall, apportionment is used to divide up
resources (human or otherwise) in as fair a way as possible.

Basic Concepts of Apportionment

The seats are the people or items that are to be shared equally. The states are the
parties that will receive a proportional share of the seats.

The first step in any apportionment problem is to calculate the standard divisor. This is
the ratio of the total population to the number of seats. It tells us how many people are
represented by each seat.

The standard divisor is SD = total population/ no. of seats.

The next step is to find the standard quota for each state. This is the exact number of seats that
should be allocated to each state if decimal values were possible.

The standard quota is SQ = standard divisor/ state population


85
SUBJECT: GEED 10053: MATHEMATICS IN THE MODERN WORLD
PREPARED BY: Assoc. Prof. CHERRY E. ANGELES & WELFREDO B. QUEREMIT, LPT
For further reading about this lesson click :

https://www.coconino.edu/resources/files/pdfs/academics/arts-and-
sciences/MAT142/Chapter_9_Apportionment.pdf

Activities/Assessments:

Solve the problems below:

1.
Child Allan Betty Connie Douglas Ellie Total
Minutes worked 150 78 173 204 295 900

Kitchen Capitalism Mom has 50 pieces of identical candy to split among her 5 children. She decides that
each child will earn a proportion of the candy based on how many minutes of chores they did during the
week. How many pieces of candy should go to Alan?

2. Your college campus is broken into five sections. The board of trustees has recently approved the
installation of 70 new emergency blue lights. The lights will be apportioned based on the area of each
section. That is, the larger the section, the more lights that it will receive. The table below gives the area,
in acres, of each section of campus.

Campus Main North East Campus West South


Section Campus Campus Campus Campus
Area (acres) 516 674 378 392 140

Identify:
The states: ____________________________________________________
The seats: _____________________________________________________
The populations: ________________________________________________

ASSIGNMENT

Solve the problems below:

1. Parador is a new republic in Central America and consists of six states, which we call A, B, C,
D, E, and F for simplicity. There are 250 seats in Parador’s Congress. What is the “correct
apportionment”?

State A B C D E F Total
Population 1,646,000 6,936,000 154,000 2,091,000 685,000 988,000 12,500,000

86
SUBJECT: GEED 10053: MATHEMATICS IN THE MODERN WORLD
PREPARED BY: Assoc. Prof. CHERRY E. ANGELES & WELFREDO B. QUEREMIT, LPT
References:

https://www.avon-
schools.org/cms/lib/IN01001885/Centricity/Domain/3488/FA%20Ch%204%20Notes%20-
%20Leahy.pdf

UNIT 2: Introduction to Voting

Mathematics of Voting

Voting, from a mathematical perspective, is the process of aggregating the


preferences of individuals in a way that attempts to describe the preferences of a whole group.
This can be either for voting on a single best option--such as which restaurant you and your
friends would like to go to--or determining who should be let in to a small group of decision
makers--such as deciding how many seats should go to students, faculty, and administration on
a university's decision board.

Below, one surprisingly strong voting method and several related paradoxes in the
mathematics of voting theory are discussed.
1. Condorcet's Paradox

The Condorcet method is a robust method for determining aggregate preference from
individual preference which works by taking every possible comparison between two
choices, finding the victor, and then chaining the pairwise results together to form an
overall ordering.

2. Arrow’s Impossibility Theorem

Three desirable features for a voting system are as follows:

 Unanimity: If everyone prefers AA to BB, AA should win.


 No Dictators: There should not be anyone whose individual preferences
always determine who wins.
 Independence of Irrelevant Alternatives (IIA): Adding extra options should
not make existing relations change. That is, if A≥B, adding option C should
not make B≥A.
Independence of irrelevant alternatives is an important criterion since ideally a voting
system will not be susceptible to strategic voting, in which voters will rank options in

87
SUBJECT: GEED 10053: MATHEMATICS IN THE MODERN WORLD
PREPARED BY: Assoc. Prof. CHERRY E. ANGELES & WELFREDO B. QUEREMIT, LPT
ways that do not reveal their true preferences so as to attempt to make sure their top
choice is elected.

3. Apportionment Paradox

Whereas Arrow's theorem states impossibility results for the preferences over a set of
particular candidates, the apportionment paradox makes similar statements for fairly
dividing up a number of discrete seats to different groups.

Representative democracy is a form of government in which, instead of having every


individual vote on every issue (direct democracy), individuals instead elect a small
number of representatives to vote in their interests. The apportionment paradox is an
impossibility theorem for choosing the number of representative seats to be assigned to
each group. In countries with party-list proportional representation, the groups are
political parties. Each party gets a number of seats that is a function of the number of
people voting for that party. In the United States, the groups are states. The number of
seats they get in the House of Representatives is a function of the population of the state.
Residence in a state is a “vote” for a seat for that state (in a sense separate from the
actual votes for what candidates will fill the seats given to the state).

3. d'Hondt's Method or Jefferson's Method

Another approach, known as d'Hondt's method or Jefferson's method, is to keep


reducing the value of D so that when all the quotas are rounded down, the quotas add
together to the correct number of seats.

The current method in use by the US House of Representative is known as Huntington-


Hill. Each Qstate is temporarily rounded down (call this value n), but then this value is

compared against the geometric mean of n and n + 1:

If Qstate is greater than the geometric mean, then Qstate is rounded up; otherwise it is
​ ​

rounded down.

Just like d'Hondt's method, DD may need to be adjusted to ensure all the quotas
together add to the correct number of seats.

4. Gerrymandering

In the same way that a nation is divided into states, states are divided into districts, each
of which votes on a particular candidate. Candidates are elected by counting the number
of districts they win, under the assumption that winning the most districts is the same as
winning the overall vote. However, it is possible for one candidate to win in most districts

88
SUBJECT: GEED 10053: MATHEMATICS IN THE MODERN WORLD
PREPARED BY: Assoc. Prof. CHERRY E. ANGELES & WELFREDO B. QUEREMIT, LPT
while still losing the popular vote. Gerrymandering refers to the practice of purposefully
redrawing district lines so that one candidate is more likely to win.

Also present in voting is Simpson's paradox in statistics, which says it is possible for
variables to be positively correlated in subgroups despite being negatively correlated
overall.

For further reading about this lesson click:

https://brilliant.org/wiki/mathematics-of-voting/

Activities/Assessments:

Solve the problems below:

1. There are three groups A, B, and C that need to divide up a board with 5 seats on it. A has
500 members, B has 300 members, and C has 200 members. If seats are assigned using
Huntington – Hill Method, how many seats go to group A.?

2. It is election night in a city if several million people and an exit poll of 382 voters shows that
Mr. Fake Smile is leading Ms. Empty Promises 52% to 48% in the run for mayor. Can election
be called with 95% certainty?

ASSIGNMENT

Click this link to answer your assignment

https://brilliant.org/problems/surviving-elimination/

References:

https://marinmathcircledotorg.files.wordpress.com/2015/12/mmcadv-20120111-votinglecture-
ernestodiaz.pdf
https://college.cengage.com/mathematics/bello/topics/9e/assets/students/14se.pdf

UNIT 3: Weighted Voting System

There are some types of elections where the voters do not all have the same amount of power.
This happens often in the business world where the power that a voter possesses may be
based on how many shares of stock he/she owns. In this situation, one voter may control the
equivalent of 100 votes where other voters only control 15 or 10 or fewer votes. Therefore, the
amount of power that each voter possesses is different. Another example is in how the
President of the United States is elected. When a person goes to the polls and casts a vote for
President, he or she is actually electing who will go to the Electoral College and represent that
state by casting the actual vote for President. Each state has a certain number of Electoral
89
SUBJECT: GEED 10053: MATHEMATICS IN THE MODERN WORLD
PREPARED BY: Assoc. Prof. CHERRY E. ANGELES & WELFREDO B. QUEREMIT, LPT
College votes, which is determined by the number of Senators and number of Representatives
in Congress. Some states have more Electoral College votes than others, so some states have
more power than others. How do we determine the power that each state possesses?

To figure out power, we need to first define some concepts of a weighted voting system. The
individuals or entities that vote are called players. The notation for the players is P1, P2, P3,…,
PN, where N is the number of players. Each player controls a certain number of votes, which are
called the weight of that player. The notation for the weights is w1, w2, w3,…, wN where w1w1 is
the weight of P1, w2 is the weight of P2, etc. In order for a motion to pass, it must have a
minimum number of votes. This minimum is known as the quota. The notation for quota is q.
The quota must be over half the total weights and cannot be more than total weight. In other
words:

The way to denote a weighted voting system is [q: w1, w2, w3,…, wN]

Example:

A company has 5 shareholders. Ms. Lee has 30% ownership, Ms. Miller has 25%, Mr.
Matic has 22% ownership, Ms. Pierce has 14%, and Mr. Hamilton has 9%. There is a
motion to decide where best to invest their savings. The company’s by-laws define the
quota as 58%. What does this voting system look like?

Solution

Treating the percentages of ownership as the votes, the system looks like: 58:30,25,22,14,9]

For further readings of this lesson click:


https://math.libretexts.org/Bookshelves/Applied_Mathematics/Book%3A_College_Mathematics_
for_Everyday_Life_(Inigo_et_al)/07%3A_Voting_Systems/7.02%3A_Weighted_Voting
http://www.opentextbookstore.com/mathinsociety/2.4/WeightedVoting.pdf

Activities/Assessments:

Solve the following problems:

1. How many players are there in the weighted voting system 20:7,5,4,4,2,2,2,1,1]?

2. What is the quota in the weighted voting system 20:7,5,4,4,2,2,2,1,1]?

3. What is the total number of votes in the weighted voting system 20:7,5,4,4,2,2,2,1,1]?

ASSIGNMENT

90
SUBJECT: GEED 10053: MATHEMATICS IN THE MODERN WORLD
PREPARED BY: Assoc. Prof. CHERRY E. ANGELES & WELFREDO B. QUEREMIT, LPT
Solve the following problems:

1. In the weighted voting system 10:11,4,2,1,1], specify the dictator(s) and the dummy (or
dummies).

2. In the weighted voting system 11:6,4,3,2,1], is there any one who has a veto power? If
so, who is it and why is it?

References:

https://math.libretexts.org/Bookshelves/Applied_Mathematics/Book%3A_College_Mathematics_
for_Everyday_Life_(Inigo_et_al)/07%3A_Voting_Systems/7.02%3A_Weighted_Voting
http://www.opentextbookstore.com/mathinsociety/2.4/WeightedVoting.pdf

REFERENCES

https://astrofella.wordpress.com/2019/03/31/natures-numbers-ian-stewart/

https://mathematicsinourlife.wordpress.com/

https://en.wikipedia.org/wiki/Patterns_in_nature

https://www.livescience.com/37470-fibonacci-sequence.html

http://home.windstream.net/okrebs/page131.html#:~:text=A%20sequence%20is%20a%20set,b
etween%20two%20consecutive%20terms%20constant.

https://www.mheducation.com/highered/product/math-our-world-sobecki/M9781259969690.html

https://www.researchgate.net/publication/315712910_The_Symbolic_Language_of_Mathematic
s#:~:text=Mathematics%20is%20written%20in%20a,facts%20are%20expressed%20in%20sym
bolism.

http://www.onemathematicalcat.org/pdf_files/LANG1.pdf

http://www.onemathematicalcat.org/algebra_book/online_problems/exp_vs_sen.htm

https://www.toppr.com/guides/business-mathematics-and-statistics/sets-relations-and-
functions/basic-definitions-and-concepts/

http://www-math.ucdenver.edu/~wcherowi/courses/m3000/lecture2.pdf

https://www.investopedia.com/terms/s/statistics.asp#:~:text=Statistics%20is%20a%20form%20o
f,Mean
91
SUBJECT: GEED 10053: MATHEMATICS IN THE MODERN WORLD
PREPARED BY: Assoc. Prof. CHERRY E. ANGELES & WELFREDO B. QUEREMIT, LPT
https://brilliant.org/wiki/linear-programming/

https://www.pearson.com/content/dam/one-dot-com/one-dot-com/us/en/higher-ed/en/products-
services/course-products/lial-applied-mathematics-info/pdf/LGR-Finite-Ch5.pdf

https://finance.zacks.com/difference-between-stocks-bonds-mutual-funds-2329.html

https://www.lorenzfinancialservices.com/investments/difference-between-stocks-bonds-mutual-
funds/

https://www.coconino.edu/resources/files/pdfs/academics/arts-and-
sciences/MAT142/Chapter_9_Apportionment.pdf

https://www.avon-
schools.org/cms/lib/IN01001885/Centricity/Domain/3488/FA%20Ch%204%20Notes%20-
%20Leahy.pdf

https://brilliant.org/wiki/mathematics-of-voting/

https://marinmathcircledotorg.files.wordpress.com/2015/12/mmcadv-20120111-votinglecture-
ernestodiaz.pdf

https://college.cengage.com/mathematics/bello/topics/9e/assets/students/14se.pdf

https://math.libretexts.org/Bookshelves/Applied_Mathematics/Book%3A_College_Mathematics_
for_Everyday_Life_(Inigo_et_al)/07%3A_Voting_Systems/7.02%3A_Weighted_Voting

http://www.opentextbookstore.com/mathinsociety/2.4/WeightedVoting.pdf

Book(s):

Mathematics in the Modern World by Aufmann, Richard, et. Al.

Mathematics of Investment by Cervillon, Carmelita C., et. al.

92
SUBJECT: GEED 10053: MATHEMATICS IN THE MODERN WORLD
PREPARED BY: Assoc. Prof. CHERRY E. ANGELES & WELFREDO B. QUEREMIT, LPT

You might also like